Les hormones humaines sont les principales. Hormones féminines

💖 Vous aimez ça ? Partagez le lien avec vos amis

1. Quelles substances sont appelées hormones ? Quelles sont leurs principales propriétés ?

Les hormones - composants chimiques, qui ont une activité biologique élevée, sont sécrétées par les glandes endocrines.

Propriétés des hormones :

  • produit en petites quantités;
  • caractère d'action distant (les organes et les systèmes sur lesquels agissent les hormones sont situés loin du lieu de leur formation, les hormones sont donc distribuées dans tout le corps par la circulation sanguine) ;
  • sont stockés pendant une longue période état actif;
  • spécificité d'action stricte;
  • activité biologique élevée;
  • réguler les processus métaboliques, assurer la constance de la composition de l’environnement, influencer la croissance et le développement des organes et assurer la réponse de l’organisme à l’influence de l’environnement extérieur.

Selon leur nature chimique, les hormones sont divisées en trois groupes : les polypeptides et les protéines (insuline) ; acides aminés et leurs dérivés (thyroxine, adrénaline) ; stéroïdes (hormones sexuelles).

Si une quantité accrue d’hormones est formée et libérée dans le sang, il s’agit d’un hyperfonctionnement. Si la quantité d’hormones produites et libérées dans le sang diminue, il s’agit alors d’un hypofonctionnement.

2. Quelles glandes produisent des hormones ? Nomme les. Quel effet les hormones de ces glandes ont-elles sur le corps ?

La glande thyroïde est située dans le cou, devant le larynx, et produit des hormones riches en iode - thyroxine, etc. Elles stimulent le métabolisme de l'organisme. Le niveau de consommation d'oxygène par les organes et tissus du corps dépend de leur quantité dans le sang, c'est-à-dire les hormones glande thyroïde stimuler les processus oxydatifs dans les cellules. De plus, ils régulent l’eau, les protéines, les graisses, les glucides, le métabolisme minéral, la croissance et le développement de l’organisme. Avoir un effet sur les fonctions de la centrale système nerveux et plus haut activité nerveuse. Le manque d'hormone dans l'enfance conduit au crétinisme (croissance, sexualité et développement mental, les proportions du corps sont perturbées). En cas d'hypofonctionnement, un adulte développe un myxœdème (diminution du métabolisme, obésité, diminution de la température corporelle, apathie). Quand l'hyperfonctionnement survient chez l'adulte Maladie de Graves(hypertrophie de la glande thyroïde, développement d'un goitre, yeux exorbités, augmentation du métabolisme, excitabilité accrue système nerveux).

Glandes surrénales. Petits corps au-dessus des reins. Ils sont constitués de deux couches : externe (corticale) et interne (cérébrale). La substance externe produit des hormones qui régulent le métabolisme (sodium, potassium, protéines, glucides, graisses) et des hormones sexuelles (déterminent le développement des caractères sexuels secondaires). Avec une fonction insuffisante du cortex surrénalien, une maladie se développe, appelée maladie du bronze. La peau acquiert une couleur bronze, une fatigue accrue, une perte d'appétit et des nausées sont observées. Avec l'hyperfonctionnement des glandes surrénales, il y a une augmentation de la synthèse des hormones sexuelles. Dans le même temps, les caractères sexuels secondaires changent. Par exemple, les femmes développent une moustache, une barbe, etc.

La substance interne produit les hormones adrénaline et noradrénaline. L'adrénaline accélère la circulation sanguine, augmente la fréquence cardiaque, mobilise toutes les forces du corps lors des situations stressantes, augmente la glycémie (dégrade le glycogène). La quantité d’adrénaline est sous le contrôle du système nerveux central ; elle ne manque pas. En excès, il augmente la fréquence cardiaque, rétrécit vaisseaux sanguins. La norépinéphrine ralentit la fréquence cardiaque.

Pancréas. Est dans cavité abdominale corps, sous l'estomac. C'est une glande sécrétion mixte, possède des canaux excréteurs et sécrète des enzymes impliquées dans la digestion. Les cellules individuelles du pancréas libèrent des hormones dans le sang. Un groupe de cellules produit l’hormone glucagon, qui aide à convertir le glycogène hépatique en glucose, provoquant ainsi une augmentation de la glycémie. D'autres cellules produisent de l'insuline. C'est la seule hormone qui abaisse la glycémie (favorise la synthèse du glycogène dans les cellules hépatiques). En cas d'insuffisance de la fonction pancréatique, il se développe diabète. Dans le même temps, le taux de sucre dans le sang augmente. Les glucides ne sont pas retenus dans l'organisme, mais sont excrétés dans l'urine sous forme de glucose.

Les glandes sexuelles – testicules chez l’homme et ovaires chez la femme – appartiennent également aux glandes à sécrétion mixte. En raison de fonction exocrine des spermatozoïdes et des ovules se forment. La fonction endocrinienne est associée à la production d'hormones sexuelles mâles et femelles, qui régulent le développement des caractères sexuels secondaires. Ils influencent la formation corporelle, le métabolisme et le comportement sexuel. Les androgènes sont produits dans les testicules. Ils stimulent le développement des caractères sexuels secondaires caractéristiques de l'homme (pousse de la barbe, de la moustache, développement musculaire, etc.), augmentent le métabolisme basal et sont nécessaires à la maturation des spermatozoïdes.

Les ovaires produisent des hormones sexuelles féminines - les œstrogènes, sous l'influence desquelles se produit la formation de caractères sexuels secondaires caractéristiques des femmes (forme du corps, développement des glandes mammaires, etc.) Matériel du site

Pituitaire. Il est situé sous le pont du cerveau et se compose de trois lobes : antérieur, intermédiaire et postérieur. Le lobe antérieur sécrète l'hormone de croissance, qui affecte la croissance des os en longueur, accélère les processus métaboliques, entraîne une croissance accrue et une augmentation du poids corporel. Une carence hormonale entraîne le nanisme, mais les proportions corporelles et le développement mental ne sont pas altérés. L'hyperfonctionnement dans l'enfance conduit au gigantisme (les enfants ont des membres longs et ne sont pas assez forts physiquement) ; chez l'adulte, il y a une acromégalie (la taille de la main, du pied, partie avant crâne, nez, lèvres, menton). L'hypofonctionnement chez l'adulte entraîne des modifications du métabolisme : soit une obésité, soit une perte de poids brutale.

Le lobe intermédiaire de l'hypophyse sécrète une hormone qui affecte la pigmentation de la peau.

Le lobe postérieur est formé de tissu nerveux. Il ne synthétise pas d'hormones. Les substances biologiquement actives produites par les noyaux de l'hypothalamus sont transportées vers le lobe postérieur de l'hypophyse. L'un d'eux affecte sélectivement les contractions des muscles lisses de l'utérus et la sécrétion des glandes mammaires. Autres augmentations pression artérielle et retarde l'excrétion urinaire. Lorsque la quantité de cette substance diminue, la miction augmente jusqu'à 10 à 20 litres. par jour. Cette maladie est appelée diabète insipide.

Vous n'avez pas trouvé ce que vous cherchiez ? Utilisez la recherche

Sur cette page, vous trouverez du matériel sur les sujets suivants :

  • nommer les glandes temporaires, quelles hormones elles produisent et quelle est leur signification
  • résumé des hormones
  • nommer les principales propriétés des hormones
  • que sont les hormones et quelles sont leurs propriétés ?
  • hormones brièvement à leur sujet

La particularité des hormones est qu'une personne a besoin d'une petite quantité de chaque substance, mais en même temps elles sont toutes d'une grande importance pour le corps.

Les principales fonctions des hormones : régulation des processus métaboliques, croissance cellulaire, développement des organes. Produit à l'aide du système endocrinien dont la structure comprend :

  • pituitaire;
  • hypothalamus;
  • thyroïde et pancréas ;
  • glandes surrénales

En cas de pannes dans système hormonal, la personne commence à souffrir de manifestations diverses maladies.

Caractéristiques générales

Combien de types d’hormones le corps humain produit-il ? Les médecins comptent environ 100 variétés d'hormones basiques et plus d'une douzaine d'hormones activatrices. Après production, ils sont affichés dans sang et sont dirigés vers l'organe ou le tissu requis, où ils affectent chaque cellule. Les composants protéiques sont capables de fonctionner à la surface des membranes cellulaires, tandis que les composants graisseux pénètrent à l'intérieur et affectent les organites.

Selon leur propre propriétés chimiques sont divisés en plusieurs substances :

  • protéines;
  • dérivés d'acides aminés;
  • des peptides ;
  • les graisses;
  • des stéroïdes.

Ensemble, ils contribuent à la maturation physique, mentale et sexuelle d'une personne. Et grâce à ces substances, le corps s'adapte facilement au monde extérieur changeant et maintient la constance de son environnement interne. Chaque hormone a sa propre structure chimique et ses propres propriétés physiques.

Toutes les hormones produites par le corps peuvent être divisées en 5 groupes :

  • croissance et régulation (glande pituitaire);
  • reproducteur (ovaires et testicules) :
  • stress (médullosurrénale);
  • corticostéroïdes (cortex surrénalien);
  • métabolique (pancréas et glandes thyroïde).

Les hormones activatrices ne sont incluses dans aucun des groupes ci-dessus. Ils n'ont aucune influence directe sur corps humain. Ces substances stimulent la synthèse d'hormones essentielles. Synthétisé par l'hypothalamus et les lobes antérieurs.

Croissance et réglementation

La concentration d'hormones sexuelles dans le corps féminin n'est pas constante. Des sauts brusques se produisent sous l'influence des phases du cycle menstruel. Le plus Grands changements V fond hormonal surviennent pendant la grossesse.

Stressant

Ces hormones sont produites dans l’organisme par les glandes surrénales. Ils influencent processus métaboliques et l'adaptation humaine aux conditions changeantes environnement. Grâce à eux nous pouvons combattre le stress et accepter décisions importantes dans des conditions extrêmes.

Dopamine

Ou, en d’autres termes, « l’hormone de la joie ». C'est lui qui aide une personne à ressentir un sentiment de plaisir et d'euphorie. Le processus de production est activé dans des situations spécifiques : lorsqu'une personne aime un certain type d'activité. En même temps, le cerveau essaie de se souvenir de ces sensations et oblige la personne à y revenir encore et encore. La quantité de l'hormone peut augmenter dans des situations stressantes, et même lorsque en état de choc(y compris la douleur).

Symptômes de carence :

  • manque d'émotions;
  • indifférence à tout ce qui arrive ;
  • fatigabilité rapide;
  • forte envie de pleurer.

Symptômes d'excès :

  • respiration et fréquence cardiaque rapides;
  • une grande poussée d'énergie;
  • activité accrue.

Une diminution conduit à la dépression, qui à son tour peut provoquer l'obésité, fatigue chronique et d'autres maladies.

Adrénaline

C'est une hormone du stress. Cela aide à « se ressaisir » dans une situation stressante. Il peut atténuer la douleur causée par les blessures, bloquer la peur et augmenter l’endurance.

À mesure que l'adrénaline est libérée dans le sang, la fréquence cardiaque augmente, pression artérielle, la respiration, qui permet de saturer les muscles en oxygène et de les utiliser à leur plein potentiel. Cette substance augmente également la période d’éveil et accélère la réaction. Combien de temps dure l'adrénaline ? Les scientifiques ont calculé que cela prend environ 5 minutes.

Un déséquilibre hormonal peut entraîner les troubles mentaux, hypertension, épuisement, maladie rénale.

Cortisol

Cette substance régule le charbon échange d'eau. Son quantité maximale produit en heures du matin. Le montant minimum s'effectue le soir.

Le cortisol est également libéré dans le sang lors de situations stressantes. Il aide le corps humain à se mobiliser en réduisant l'absorption du calcium et en modifiant le métabolisme, rendant ainsi le glucose plus disponible. Lorsqu'il y a un manque de sang, une personne commence à se sentir irritable, souffre de maux de tête et de vertiges, perd l'appétit et perturbe le fonctionnement du tractus gastro-intestinal.

L’excès d’hormones provoque :

  • obésité;
  • insomnie;
  • diminution des défenses immunitaires;
  • diminution des niveaux de testostérone dans le corps.

Tout cela peut provoquer de nombreuses maladies : diabète, ostéoporose et maladies cardiovasculaires.

Corticostéroïdes

Maintient l’équilibre minéral dans le corps. Les hormones de ce groupe sont produites dans le cortex surrénalien. Leur fonctionnalité ne se limite pas à un seul organe ou tissu spécifique.

Ils régulent tous les processus métaboliques du corps, maintiennent une constante composition minérale le sang, favorisent l'élimination des substances en excès. Ils sont également utilisés à des fins médicales :

  • pour le traitement de l'hépatite virale ;
  • prévention de l'arthrite;
  • traitement de l'arthrose;
  • la prévention l'asthme bronchique.

Échange

Ce groupe comprend différents types d'hormones, mais elles sont toutes unies par une fonction commune : réguler les processus métaboliques du corps. Ils sont synthétisés à l'aide du pancréas, de la thyroïde et des glandes parathyroïdes et régulent le processus de miction.

L'équilibre hormonal du corps humain est la garantie de son plein épanouissement.

Ces substances sont très importantes pendant l’enfance et l’adolescence, car elles stimulent la croissance et la formation du corps. Troubles hormonaux V corps d'enfant très difficile à compenser et peut conduire à des conséquences irréversibles. L'état des hormones dans le corps affecte l'état de tous les organes.

Les hormones thyroïdiennes méritent une attention particulière.

Avec leur carence, le développement physique et mental est inhibé. De plus, les hormones thyroïdiennes interagissent étroitement avec d’autres hormones. Un exemple frappant de ce processus est le lien avec la somatotropine, responsable de la croissance du corps. Cette hormone est indispensable dans le corps d'un adolescent.

Symptômes des troubles thyroïdiens :

  • problèmes de poids - être en surpoids ou en surpoids ;
  • ralentissement de la croissance ;
  • larmoiement et irritabilité;
  • cou enflé et globes oculaires hypertrophiés ;
  • peau pâle;
  • fatigue accrue;
  • diminution de l'activité mentale.

Si ces signes apparaissent, vous devriez subir un test d’hormones thyroïdiennes. S'il n'y a pas de caractères sexuels secondaires chez un adolescent âgé de 12-14 ans, il faut vérifier. Et aussi dans enfance Vous pouvez faire un test de glycémie et diagnostiquer la présence de diabète.

Le contenu de l'article

LES HORMONES, composés organiques produits par certaines cellules et conçus pour contrôler, réguler et coordonner les fonctions du corps. Les animaux supérieurs ont deux systèmes de régulation à l'aide desquels le corps s'adapte aux constantes internes et changements externes. L'un d'eux est le système nerveux, qui transmet rapidement des signaux (sous forme d'impulsions) à travers un réseau de nerfs et de cellules nerveuses ; l'autre est endocrinien, qui effectue une régulation chimique à l'aide d'hormones transportées dans le sang et ayant un effet sur les tissus et organes éloignés du lieu de leur libération. Système chimique la communication interagit avec le système nerveux ; Ainsi, certaines hormones fonctionnent comme médiateurs (messagers) entre le système nerveux et les organes qui répondent à l'influence. Ainsi, la distinction entre coordination neuronale et chimique n’est pas absolue.

L'action physiologique des hormones vise à : 1) fournir de l'humour, c'est-à-dire réalisée par le sang, régulation processus biologiques; 2) maintenir l'intégrité et la constance de l'environnement interne, interaction harmonieuse entre les composants cellulaires de l'organisme ; 3) régulation des processus de croissance, de maturation et de reproduction.

Les hormones régulent l’activité de toutes les cellules du corps. Ils affectent l'acuité mentale et la mobilité physique, le physique et la taille, déterminent la croissance des cheveux, le ton de la voix, la libido et le comportement. Grâce au système endocrinien, une personne peut s'adapter à de fortes fluctuations de température, à un excès ou à un manque de nourriture, à des problèmes physiques et stress émotionnel. L'étude de l'action physiologique des glandes endocrines a permis de révéler les secrets de la fonction sexuelle et le miracle de l'accouchement, ainsi que de répondre à la question de savoir pourquoi certaines personnes sont grandes et d'autres petites, certaines sont grosses, d'autres sont minces. , certains sont lents, d'autres sont agiles, certains sont forts, d'autres sont faibles.

DANS en bonne condition il existe un équilibre harmonieux entre l'activité des glandes endocrines, l'état du système nerveux et la réponse des tissus cibles (tissus ciblés). Toute violation de chacun de ces liens entraîne rapidement des écarts par rapport à la norme. Excessif ou production insuffisante Les hormones provoquent diverses maladies accompagnées de profonds changements chimiques dans le corps.

L'endocrinologie étudie le rôle des hormones dans la vie de l'organisme et la physiologie normale et pathologique des glandes endocrines. Elle n’est apparue comme discipline médicale qu’au XXe siècle, mais les observations endocrinologiques sont connues depuis l’Antiquité. Hippocrate croyait que la santé et le tempérament humains dépendaient de substances humorales spéciales. Aristote a attiré l'attention sur le fait qu'un veau castré, en grandissant, diffère par son comportement sexuel d'un taureau castré en ce sens qu'il n'essaie même pas de grimper sur une vache. De plus, la castration est pratiquée depuis des siècles à la fois pour apprivoiser et domestiquer les animaux et pour transformer les humains en esclaves obéissants.

Que sont les hormones ?

Selon la définition classique, les hormones sont des produits de sécrétion des glandes endocrines qui sont libérés directement dans la circulation sanguine et ont une activité physiologique élevée. Les principales glandes endocrines des mammifères sont l'hypophyse, la thyroïde et glandes parathyroïdes, cortex surrénalien, médullosurrénale, tissu des îlots pancréatiques, gonades (testicules et ovaires), placenta et sites de production d'hormones tube digestif. Le corps synthétise également certains composés ayant des effets semblables à ceux des hormones. Par exemple, des études sur l’hypothalamus ont montré qu’un certain nombre de substances qu’il sécrète sont nécessaires à la libération des hormones hypophysaires. Ces « facteurs de libération », ou libérines, ont été isolés de diverses régions de l'hypothalamus. Ils pénètrent dans l’hypophyse par un système de vaisseaux sanguins reliant les deux structures. Étant donné que l'hypothalamus n'est pas une glande dans sa structure et que les facteurs de libération ne pénètrent apparemment que dans l'hypophyse très proche, ces substances sécrétées par l'hypothalamus ne peuvent être considérées comme des hormones qu'avec une compréhension large de ce terme.

Il existe d'autres problèmes liés à la détermination des substances qui doivent être considérées comme des hormones et des structures qui doivent être considérées comme des glandes endocrines. Il a été démontré de manière convaincante que des organes tels que le foie peuvent extraire du sang circulant des substances hormonales physiologiquement inactives ou totalement inactives et les convertir en hormones puissantes. Par exemple, le sulfate de déhydroépiandrostérone, une substance peu active produite par les glandes surrénales, est convertie dans le foie en testostérone, une hormone sexuelle masculine très active sécrétée en grande quantité par les testicules. Cela prouve-t-il pour autant que le foie est un organe endocrinien ?

D'autres questions sont encore plus difficiles. Les reins sécrètent l'enzyme rénine dans la circulation sanguine qui, par l'activation du système angiotensine (ce système provoque une dilatation des vaisseaux sanguins), stimule la production de l'hormone surrénale aldostérone. La régulation de la libération d'aldostérone par ce système est très similaire à la façon dont l'hypothalamus stimule la libération de l'hormone hypophysaire ACTH (hormone adrénocorticotrope ou corticotropine), qui régule la fonction surrénale. Les reins sécrètent également de l'érythropoïétine - substance hormonale, stimulant la production de globules rouges. Le rein peut-il être classé parmi les organes endocriniens ? Tous ces exemples prouvent que la définition classique des hormones et des glandes endocrines n’est pas assez complète.

Transport d'hormones.

Les hormones, une fois dans la circulation sanguine, doivent se déplacer vers les organes cibles appropriés. Le transport des hormones de haut poids moléculaire (protéines) a été peu étudié en raison du manque de données précises sur masse moléculaire Et structure chimique beaucoup d'entre eux. Les hormones de poids moléculaire relativement faible, telles que les hormones thyroïdiennes et stéroïdes, se lient rapidement aux protéines plasmatiques, de sorte que la teneur en hormones du sang est réduite. formulaire associé plus élevé qu'en gratuit ; ces deux formes sont en équilibre dynamique. Exactement hormones gratuites présentent une activité biologique et il a été clairement démontré qu'ils sont extraits du sang par les organes cibles dans un certain nombre de cas.

L’importance de la liaison des hormones aux protéines dans le sang n’est pas entièrement claire. On pense qu'une telle liaison facilite le transport de l'hormone ou protège l'hormone d'une perte d'activité.

Action des hormones.

Les hormones individuelles et leurs principaux effets sont présentés ci-dessous dans la section « Principales hormones humaines ». En général, les hormones agissent sur des organes cibles spécifiques et provoquent des changements physiologiques importants dans ceux-ci. Une hormone peut avoir plusieurs organes cibles et les changements physiologiques qu’elle provoque peuvent affecter toute une gamme de fonctions corporelles. Par exemple, maintenir niveau normal La glycémie – et elle est largement contrôlée par les hormones – est importante pour le fonctionnement de tout le corps. Les hormones agissent parfois ensemble ; Ainsi, l’effet d’une hormone peut dépendre de la présence d’une ou plusieurs autres hormones. L’hormone de croissance, par exemple, est inefficace en l’absence d’hormone thyroïdienne.

L'action des hormones au niveau cellulaire s'effectue par deux mécanismes principaux : les hormones qui ne pénètrent pas dans la cellule (généralement hydrosolubles) agissent à travers des récepteurs sur la membrane cellulaire, et les hormones qui traversent facilement la membrane (liposolubles) agissent via des récepteurs situés dans le cytoplasme de la cellule. Dans tous les cas, seule la présence d'une protéine réceptrice spécifique détermine la sensibilité de la cellule à une hormone donnée, c'est-à-dire fait d'elle une cible. Le premier mécanisme d'action, étudié en détail à l'aide de l'exemple de l'adrénaline, est que l'hormone se lie à ses récepteurs spécifiques à la surface des cellules ; la liaison déclenche une série de réactions qui aboutissent à la formation de ce qu'on appelle. des seconds messagers qui ont un effet direct sur le métabolisme cellulaire. De tels médiateurs sont généralement l'adénosine monophosphate cyclique (AMPc) et/ou des ions calcium ; ces derniers sont libérés des structures intracellulaires ou pénètrent dans la cellule depuis l'extérieur. Les ions AMPc et calcium sont utilisés pour transmettre des signaux externes aux cellules d’une grande variété d’organismes à tous les niveaux de l’échelle évolutive. Cependant, certains récepteurs membranaires, notamment les récepteurs de l’insuline, agissent davantage raccourci: ils pénètrent de part en part dans la membrane, et lorsqu'une partie de leur molécule se lie à une hormone à la surface de la cellule, l'autre partie commence à fonctionner comme une enzyme active du côté tourné vers l'intérieur de la cellule ; cela garantit la manifestation de l'effet hormonal.

Le deuxième mécanisme d'action - via les récepteurs cytoplasmiques - est caractéristique des hormones stéroïdes (hormones surrénales et sexuelles), ainsi que des hormones thyroïdiennes (T 3 et T 4). Après avoir pénétré dans la cellule contenant le récepteur correspondant, l'hormone forme avec elle un complexe hormone-récepteur. Ce complexe est activé (à l'aide de l'ATP), après quoi il pénètre dans le noyau cellulaire, où l'hormone a un effet direct sur l'expression de certains gènes, stimulant la synthèse d'ARN et de protéines spécifiques. Ce sont ces protéines nouvellement formées, généralement de courte durée, qui sont responsables des changements qui constituent l'effet physiologique de l'hormone.

Régulation de la sécrétion hormonale

réalisée par plusieurs mécanismes interconnectés. Ils peuvent être illustrés par l’exemple du cortisol, principale hormone glucocorticoïde des glandes surrénales. Sa production est régulée par un mécanisme de rétroaction qui opère au niveau de l'hypothalamus. Lorsque le taux de cortisol dans le sang diminue, l’hypothalamus sécrète de la corticolibérine, un facteur qui stimule l’hypophyse à sécréter de la corticotropine (ACTH). L’augmentation des niveaux d’ACTH stimule à son tour la sécrétion de cortisol dans les glandes surrénales et, par conséquent, le niveau de cortisol dans le sang augmente. L'augmentation du taux de cortisol supprime alors la libération de corticolibérine via un mécanisme de rétroaction - et la teneur en cortisol dans le sang diminue à nouveau.

La sécrétion de cortisol n'est pas régulée uniquement par un mécanisme de rétroaction. Par exemple, le stress provoque la libération de corticolibérine et, par conséquent, toute une série de réactions qui augmentent la sécrétion de cortisol. De plus, la sécrétion de cortisol suit un rythme circadien ; il est très élevé au réveil, mais diminue progressivement jusqu'à un niveau minimum pendant le sommeil. Les mécanismes de contrôle incluent également le taux de métabolisme hormonal et la perte d’activité. Des systèmes de régulation similaires fonctionnent pour d’autres hormones.

PRINCIPALES HORMONES HUMAINES

Hormones hypophysaires

sont décrits en détail dans l'article PITUITARY physis. Nous énumérerons ici uniquement les principaux produits de la sécrétion hypophysaire.

Hormones de l'hypophyse antérieure.

Le tissu glandulaire du lobe antérieur produit :

– l'hormone de croissance (GH), ou somatotropine, qui affecte tous les tissus du corps, augmentant leur activité anabolisante (c'est-à-dire les processus de synthèse des composants des tissus corporels et augmentant les réserves d'énergie).

– l'hormone stimulant les mélanocytes (MSH), qui favorise la production de pigment par certaines cellules de la peau (mélanocytes et mélanophores) ;

hormone stimulant la thyroïde(TSH), qui stimule la synthèse des hormones thyroïdiennes dans la glande thyroïde ;

– la prolactine, parfois appelée PRL, est une hormone qui stimule la formation des glandes mammaires et la lactation.

Hormones hypophysaires postérieures

– vasopressine et ocytocine. Les deux hormones sont produites dans l’hypothalamus mais sont stockées et libérées dans le lobe postérieur de l’hypophyse, situé en dessous de l’hypothalamus. La vasopressine maintient le tonus des vaisseaux sanguins et est hormone antidiurétique affectant le métabolisme de l’eau. L'ocytocine provoque des contractions de l'utérus et a la propriété de « libérer » le lait après l'accouchement.

Hormones thyroïdiennes et parathyroïdiennes.

La glande thyroïde est située dans le cou et est constituée de deux lobes reliés par un isthme étroit ( cm. THYROÏDE). Les quatre glandes parathyroïdes sont généralement situées par paires - sur les surfaces postérieures et latérales de chaque lobe de la glande thyroïde, bien que parfois une ou deux puissent être légèrement déplacées.

Les principales hormones sécrétées par la glande thyroïde normale sont la thyroxine (T 4) et la triiodothyronine (T 3). Une fois dans la circulation sanguine, ils se lient – ​​fermement mais de manière réversible – à des protéines plasmatiques spécifiques. Le T4 se lie plus fortement que le T3 et n’est pas libéré aussi rapidement ; il agit donc plus lentement mais dure plus longtemps. Les hormones thyroïdiennes stimulent synthèse des protéines et la pourriture nutriments avec dégagement de chaleur et d'énergie, qui se manifeste par une consommation accrue d'oxygène. Ces hormones influencent également le métabolisme des glucides et, avec d'autres hormones, régulent le taux de mobilisation des glucides libres. Les acides grasà partir du tissu adipeux. En bref, les hormones thyroïdiennes ont un effet stimulant sur les processus métaboliques. Une production accrue d'hormones thyroïdiennes provoque une thyréotoxicose et, en cas de déficit, une hypothyroïdie ou un myxœdème se produit.

Un autre composé présent dans la glande thyroïde est un stimulant thyroïdien à action prolongée. Il s’agit d’une gammaglobuline susceptible de provoquer un état d’hyperthyroïdie.

L'hormone produite par les glandes parathyroïdes est appelée hormone parathyroïdienne, ou hormone parathyroïdienne ; il maintient un taux constant de calcium dans le sang : lorsqu'il diminue, l'hormone parathyroïdienne est libérée et active le transfert du calcium des os vers le sang jusqu'à ce que le taux de calcium dans le sang revienne à la normale. Une autre hormone, la calcitonine, a l’effet inverse et est libérée lorsque les taux de calcium dans le sang sont élevés. On croyait auparavant que la calcitonine était sécrétée par les glandes parathyroïdes, mais il a maintenant été démontré qu'elle est produite dans la glande thyroïde. Une production accrue d'hormone parathyroïdienne provoque des maladies osseuses, des calculs rénaux, une calcification des tubules rénaux, et une combinaison de ces troubles est possible. Le déficit en hormone parathyroïdienne s'accompagne d'une diminution significative du taux de calcium dans le sang et se manifeste par une augmentation excitabilité neuromusculaire, spasmes et convulsions.

Hormones surrénales.

Les glandes surrénales sont de petites structures situées au-dessus de chaque rein. Ils sont constitués d’une couche externe appelée cortex et d’une partie interne appelée moelle. Les deux parties ont leurs propres fonctions et, chez certains animaux inférieurs, elles constituent des structures complètement distinctes. Chacune des deux parties des glandes surrénales joue un rôle important tant dans la santé normale que dans la maladie. Par exemple, l'une des hormones de la moelle - l'adrénaline - est nécessaire à la survie, car elle réagit à un danger soudain. Lorsque cela se produit, l'adrénaline est libérée dans le sang et mobilise les réserves de glucides pour une libération rapide de l'énergie, ce qui augmente force musculaire, provoque une dilatation des pupilles et une constriction des vaisseaux sanguins périphériques. Ainsi, les forces de réserve sont dirigées vers « la fuite ou le combat », et en outre, la perte de sang est réduite en raison de la vasoconstriction et de la coagulation sanguine rapide. L'épinéphrine stimule également la sécrétion d'ACTH (c'est-à-dire l'axe hypothalamo-hypophysaire). L'ACTH, à son tour, stimule le cortex surrénalien à libérer du cortisol, ce qui entraîne une augmentation de la conversion des protéines en glucose, nécessaire pour reconstituer les réserves de glycogène dans le foie et les muscles utilisés dans la réponse à l'anxiété.

Le cortex surrénalien sécrète trois groupes principaux d'hormones : les minéralocorticoïdes, les glucocorticoïdes et les stéroïdes sexuels (androgènes et œstrogènes). Les minéralocorticoïdes sont l'aldostérone et la désoxycorticostérone. Leur action est principalement associée au maintien de l’équilibre salin. Les glucocorticoïdes affectent le métabolisme des glucides, des protéines, des graisses ainsi que les mécanismes de défense immunologiques. Les glucocorticoïdes les plus importants sont le cortisol et la corticostérone. Les stéroïdes sexuels qui jouent un rôle auxiliaire sont similaires à ceux synthétisés dans les gonades ; il s'agit du sulfate de déhydroépiandrostérone, de la D 4 -androstènedione, de la déhydroépiandrostérone et de certains œstrogènes.

L'excès de cortisol entraîne de graves troubles métaboliques, provoquant une hypergluconéogenèse, c'est-à-dire conversion excessive des protéines en glucides. Cette pathologie, connue sous le nom de syndrome de Cushing, se caractérise par une perte de masse musculaire, tolérance réduite aux glucides, c'est-à-dire une diminution de l'apport de glucose du sang aux tissus (qui se manifeste par une augmentation anormale de la concentration de sucre dans le sang lorsqu'il provient de l'alimentation), ainsi qu'une déminéralisation des os.

La sécrétion excessive d'androgènes par les tumeurs surrénales conduit à la masculinisation. Les tumeurs surrénales peuvent également produire des œstrogènes, en particulier chez les hommes, entraînant une féminisation.

L'hypofonction (activité réduite) des glandes surrénales survient dans des cas aigus ou forme chronique. L'hypofonctionnement est provoqué par une infection bactérienne grave et à évolution rapide : elle peut endommager la glande surrénale et entraîner un choc profond. Dans la forme chronique, la maladie se développe en raison d'une destruction partielle de la glande surrénale (par exemple, par une tumeur en croissance ou la tuberculose) ou par la production d'auto-anticorps. Cette maladie, connue sous le nom de maladie d'Addison, se caractérise par une faiblesse sévère, une perte de poids, une hypotension artérielle, des troubles gastro-intestinaux, besoin accru dans le sel et la pigmentation de la peau. La maladie d'Addison, décrite en 1855 par T. Addison, est devenue la première maladie endocrinienne reconnue.

L'adrénaline et la noradrénaline sont les deux principales hormones sécrétées par la médullosurrénale. L'épinéphrine est considérée comme une hormone métabolique en raison de ses effets sur le stockage des glucides et la mobilisation des graisses. La norépinéphrine est un vasoconstricteur, c'est-à-dire il resserre les vaisseaux sanguins et augmente la tension artérielle. La médullosurrénale est étroitement liée au système nerveux ; donc, la norépinéphrine est libérée nerfs sympathiques et agit comme une neurohormone.

Une sécrétion excessive d'hormones de la médullosurrénale (hormones médullaires) se produit avec certaines tumeurs. Les symptômes dépendent de laquelle des deux hormones, l'adrénaline ou la noradrénaline, est produite en plus grande quantité, mais les plus courants sont des crises soudaines de bouffées de chaleur, de transpiration, d'anxiété, de palpitations et mal de tête et l'hypertension artérielle.

Hormones testiculaires.

Les testicules (testicules) comportent deux parties, étant des glandes à sécrétion externe et interne. En tant que glandes exocrines, elles produisent des spermatozoïdes et la fonction endocrinienne est assurée par les cellules de Leydig qu'elles contiennent, qui sécrètent des hormones sexuelles mâles (androgènes), notamment la D 4 -androstènedione et la testostérone, les principales hormone masculine. Les cellules de Leydig produisent également de petites quantités d’œstrogène (œstradiol).

Les testicules sont sous le contrôle des gonadotrophines ( voir au dessus chapitre HORMONES PITUITAIRES). La gonadotrophine FSH stimule la formation des spermatozoïdes (spermatogenèse). Sous l’influence d’une autre gonadotrophine, la LH, les cellules de Leydig libèrent de la testostérone. La spermatogenèse ne se produit que lorsqu'il y a une quantité suffisante d'androgènes. Les androgènes, en particulier la testostérone, sont responsables du développement des caractères sexuels secondaires chez l'homme.

La violation de la fonction endocrinienne des testicules se résume dans la plupart des cas à une sécrétion insuffisante d'androgènes. Par exemple, l’hypogonadisme est une diminution de la fonction testiculaire, notamment de la sécrétion de testostérone et/ou de la spermatogenèse. La cause de l'hypogonadisme peut être une maladie des testicules ou, indirectement, une défaillance fonctionnelle de l'hypophyse.

Une sécrétion accrue d'androgènes se produit dans les tumeurs à cellules de Leydig et conduit à un développement excessif des caractéristiques sexuelles masculines, en particulier chez les adolescents. Parfois, les tumeurs testiculaires produisent des œstrogènes, provoquant une féminisation. Dans le cas d'une tumeur rare des testicules - le choriocarcinome - il y a tellement de gonadotrophines chorioniques humaines produites que l'analyse quantité minimum l'urine ou le sérum donne les mêmes résultats que chez la femme enceinte. Le développement du choriocarcinome peut conduire à une féminisation.

Hormones ovariennes.

Les ovaires ont deux fonctions : développer des ovules et sécréter des hormones ( voir également REPRODUCTION HUMAINE). Les hormones ovariennes sont les œstrogènes, la progestérone et la D 4 -androstènedione. Les œstrogènes déterminent le développement des caractères sexuels secondaires féminins. L'œstrogène ovarien, l'estradiol, est produit dans les cellules du follicule en croissance, le sac qui entoure l'ovule en développement. Sous l’action de la FSH et de la LH, le follicule mûrit et se rompt, libérant l’ovule. Le follicule rompu se transforme alors en ce qu'on appelle. corps jaune, qui sécrète à la fois de l'estradiol et de la progestérone. Ces hormones, agissant ensemble, préparent la muqueuse de l'utérus (endomètre) à l'implantation d'un ovule fécondé. Si la fécondation ne se produit pas, le corps jaune subit une régression ; dans le même temps, la sécrétion d'estradiol et de progestérone s'arrête et l'endomètre se détache, provoquant les menstruations.

Bien que les ovaires contiennent de nombreux follicules immatures, au cours de chaque cycle menstruel, un seul d'entre eux mûrit et libère un ovule. Les follicules excédentaires subissent un développement inverse tout au long période de reproduction la vie de la femme. Les follicules dégénérés et les restes du corps jaune font partie du stroma, le tissu de soutien de l'ovaire. Dans certaines circonstances, des cellules stromales spécifiques sont activées et sécrètent le précurseur des hormones androgènes actives - le D 4 -androstènedione. L'activation du stroma se produit, par exemple, dans le syndrome des ovaires polykystiques, une maladie associée à une ovulation altérée. À la suite de cette activation, un excès d’androgènes est produit, ce qui peut provoquer un hirsutisme (pilosité sévère).

Une sécrétion réduite d'estradiol se produit en cas de sous-développement des ovaires. La fonction ovarienne diminue également pendant la ménopause, car l'approvisionnement en follicules est épuisé et, par conséquent, la sécrétion d'estradiol diminue, ce qui s'accompagne d'un certain nombre de symptômes, dont les plus caractéristiques sont les bouffées de chaleur. La production excessive d’œstrogènes est généralement associée aux tumeurs ovariennes. Le plus grand nombre troubles menstruels causée par un déséquilibre des hormones ovariennes et des troubles de l’ovulation.

Hormones du placenta humain.

Le placenta est une membrane poreuse qui relie l'embryon (fœtus) à la paroi de l'utérus de la mère. Il sécrète de la gonadotrophine chorionique humaine et du lactogène placentaire humain. Comme les ovaires, le placenta produit de la progestérone et de nombreux œstrogènes.

Gonadotrophine chorionique (CG).

L'implantation d'un ovule fécondé est facilitée par les hormones maternelles - l'estradiol et la progestérone. Le septième jour après la fécondation, l’embryon humain se renforce dans l’endomètre et reçoit de la nutrition provenant des tissus maternels et de la circulation sanguine. Le décollement de l'endomètre, à l'origine des menstruations, ne se produit pas car l'embryon sécrète de l'hCG, qui préserve le corps jaune : l'estradiol et la progestérone qu'il produit maintiennent l'intégrité de l'endomètre. Après l'implantation de l'embryon, le placenta commence à se développer, continuant à sécréter de l'hCG, qui atteint sa concentration la plus élevée vers le deuxième mois de la grossesse. La détermination de la concentration d'hCG dans le sang et l'urine constitue la base des tests de grossesse.

Lactogène placentaire humain (PL).

En 1962, la PL a été trouvée en concentrations élevées dans le tissu placentaire, dans le sang circulant du placenta et dans le sérum maternel. sang périphérique. La PL s’est avérée similaire, mais pas identique, à l’hormone de croissance humaine. C'est une puissante hormone métabolique. En influençant le métabolisme des glucides et des graisses, il favorise la préservation des composés contenant du glucose et de l'azote dans l'organisme de la mère et assure ainsi l'approvisionnement du fœtus. quantité suffisante nutriments; en même temps, il provoque la mobilisation des acides gras libres, source d'énergie du corps maternel.

Progestérone.

Pendant la grossesse, le taux de prégnanediol, un métabolite de la progestérone, augmente progressivement dans le sang (et l'urine) de la femme. La progestérone est sécrétée principalement par le placenta et son principal précurseur est le cholestérol présent dans le sang de la mère. La synthèse de progestérone ne dépend pas des précurseurs produits par le fœtus, à en juger par le fait qu'elle ne diminue pratiquement pas plusieurs semaines après la mort de l'embryon ; la synthèse de progestérone se poursuit également dans les cas où le fœtus a été retiré chez les patientes présentant une grossesse extra-utérine abdominale, mais le placenta a été préservé.

Œstrogènes.

Les premiers rapports faisant état de niveaux élevés d'œstrogènes dans l'urine de femmes enceintes sont apparus en 1927, et il est vite devenu évident que de tels niveaux n'étaient maintenus qu'en présence d'un fœtus vivant. Plus tard, il a été révélé qu’en cas d’anomalies fœtales associées à un développement altéré des glandes surrénales, la teneur en œstrogènes dans l’urine de la mère est considérablement réduite. Cela suggère que les hormones surrénales fœtales servent de précurseurs aux œstrogènes. D'autres études ont montré que le sulfate de déhydroépiandrostérone, présent dans le plasma fœtal, est le principal précurseur d'œstrogènes tels que l'œstrone et l'œstradiol, et que la 16-hydroxydéhydroépiandrostérone, également d'origine fœtale, est le principal précurseur d'un autre œstrogène produit par le placenta, l'œstriol. Ainsi, décharge normale Les œstrogènes présents dans l'urine pendant la grossesse sont déterminés par deux conditions : les glandes surrénales fœtales doivent synthétiser des précurseurs en quantité requise et le placenta doit les convertir en œstrogènes.

Hormones pancréatiques.

Le pancréas assure la sécrétion interne et externe. La composante exocrine (liée à la sécrétion externe) est enzymes digestives, qui, sous forme de précurseurs inactifs, entrent dans le duodénum par le canal pancréatique. La sécrétion interne est assurée par les îlots de Langerhans, qui sont représentés par plusieurs types de cellules : les cellules alpha sécrètent l'hormone glucagon, les cellules bêta sécrètent l'insuline. Le principal effet de l’insuline est d’abaisser la glycémie, principalement de trois manières : 1) inhibition de la formation de glucose dans le foie ; 2) inhibition dans le foie et les muscles de la dégradation du glycogène (un polymère de glucose, que l'organisme peut transformer en glucose si nécessaire) ; 3) stimulation de l'utilisation du glucose par les tissus. Une sécrétion insuffisante d'insuline ou sa neutralisation accrue par les auto-anticorps entraîne haut niveau la glycémie et le développement du diabète sucré. Le principal effet du glucagon est d’augmenter la glycémie en stimulant sa production dans le foie. Bien que l’insuline et le glucagon maintiennent principalement la glycémie physiologique, d’autres hormones – l’hormone de croissance, le cortisol et l’adrénaline – jouent également un rôle important.

Hormones gastro-intestinales.

Hormones du tractus gastro-intestinal - gastrine, cholécystokinine, sécrétine et pancréozymine. Ce sont des polypeptides sécrétés par la muqueuse du tractus gastro-intestinal en réponse à une stimulation spécifique. On pense que la gastrine stimule la sécrétion d'acide chlorhydrique ; la cholécystokinine contrôle la vidange de la vésicule biliaire, et la sécrétine et la pancréozymine régulent la sécrétion du suc pancréatique.

Neurohormones

- un groupe de composés chimiques sécrétés par les cellules nerveuses (neurones). Ces composés ont des propriétés semblables à celles des hormones, stimulant ou inhibant l'activité d'autres cellules ; ils comprennent les facteurs de libération mentionnés précédemment, ainsi que les neurotransmetteurs dont la fonction est de transmettre influx nerveuxà travers une étroite fente synaptique séparant l'un cellule nerveuse D'un autre. Les neurotransmetteurs comprennent la dopamine, l'épinéphrine, la noradrénaline, la sérotonine, l'histamine, l'acétylcholine et l'acide gamma-aminobutyrique.

Au milieu des années 1970, un certain nombre de nouveaux neurotransmetteurs ont été découverts et ont des effets analgésiques semblables à ceux de la morphine ; on les appelle « endorphines », c'est-à-dire "morphines internes". Les endorphines sont capables de se lier à des récepteurs spéciaux dans les structures cérébrales ; En raison de cette liaison, des impulsions sont envoyées à la moelle épinière qui bloquent la conduction des signaux de douleur entrants. L'effet analgésique de la morphine et des autres opiacés est sans doute dû à leur similitude avec les endorphines, assurant leur liaison aux mêmes récepteurs bloquant la douleur.

UTILISATIONS THÉRAPEUTIQUES DES HORMONES

Les hormones étaient initialement utilisées en cas d’insuffisance de l’une des glandes endocrines pour remplacer ou reconstituer le déficit hormonal qui en résultait. Le premier médicament hormonal efficace fut un extrait de glande thyroïde de mouton, utilisé en 1891 par le médecin anglais G. Murray pour traiter le myxœdème. Aujourd'hui, l'hormonothérapie peut compenser la sécrétion insuffisante de presque toutes les glandes endocrines ; La thérapie de remplacement effectuée après l'ablation d'une glande particulière donne également d'excellents résultats. Les hormones peuvent également être utilisées pour stimuler les glandes. Les gonadotrophines, par exemple, sont utilisées pour stimuler les gonades, notamment pour provoquer l'ovulation.

Sauf Thérapie de remplacement, les hormones et les médicaments de type hormonal sont utilisés à d’autres fins. Ainsi, la sécrétion excessive d'androgènes par les glandes surrénales dans certaines maladies est supprimée par des médicaments de type cortisone. Un autre exemple est l’utilisation d’œstrogènes et de progestérone dans les pilules contraceptives pour supprimer l’ovulation.

Les hormones peuvent également être utilisées comme agents neutralisant les effets d’autres médicaments ; dans ce cas, ils partent du fait que, par exemple, les glucocorticoïdes stimulent les processus cataboliques et les androgènes stimulent les processus anabolisants. Par conséquent, dans le contexte d'une longue cure de glucocorticoïdes (par exemple, dans le cas de polyarthrite rhumatoïde) se voient souvent prescrire en outre des agents anabolisants pour réduire ou neutraliser son effet catabolique.

Les hormones sont souvent utilisées à titre spécifique médicaments. Ainsi, l'adrénaline, qui détend les muscles lisses, est très efficace en cas de crise d'asthme bronchique. Les hormones sont également utilisées dans à des fins de diagnostic. Par exemple, lors de l'étude de la fonction du cortex surrénalien, ils ont recours à sa stimulation en injectant au patient de l'ACTH, et la réponse est évaluée par la teneur en corticostéroïdes dans l'urine ou le plasma.

Actuellement, les préparations hormonales ont commencé à être utilisées dans presque tous les domaines de la médecine. Les gastro-entérologues utilisent des hormones de type cortisone dans le traitement de l'entérite régionale ou de la colite muqueuse. Les dermatologues traitent l'acné avec des œstrogènes et certaines maladies de peau avec des glucocorticoïdes ; Les allergologues utilisent l'ACTH et les glucocorticoïdes dans le traitement de l'asthme, de l'urticaire et d'autres maladies allergiques. Les pédiatres ont recours à des substances anabolisantes lorsqu'il est nécessaire d'améliorer l'appétit ou d'accélérer la croissance d'un enfant, ainsi que fortes doses des œstrogènes pour fermer les épiphyses (parties en croissance des os) et ainsi empêcher une croissance excessive.

Lors d'une transplantation d'organe, des glucocorticoïdes sont utilisés, ce qui réduit les risques de rejet de greffe. Les œstrogènes peuvent limiter la propagation du cancer du sein métastatique chez les patientes ménopausées, et les androgènes sont utilisés dans le même but avant la ménopause. Les urologues utilisent des œstrogènes pour ralentir la propagation du cancer de la prostate. Spécialistes en médecine interne ont découvert qu'il est conseillé d'utiliser des composés de type cortisone dans le traitement de certains types de collagénose, et que les gynécologues et les obstétriciens utilisent des hormones dans le traitement de nombreux troubles non directement liés à un déficit hormonal.

HORMONES D'INVERTÉBRÉS

Les hormones des invertébrés ont été étudiées principalement chez les insectes, les crustacés et les mollusques, mais beaucoup de choses dans ce domaine restent encore floues. Parfois, le manque d'informations sur les hormones d'une espèce animale particulière s'explique simplement par le fait que cette espèce ne possède pas de glandes endocrines spécialisées et que les groupes individuels de cellules sécrétant des hormones sont difficiles à détecter.

Il est probable que toute fonction régulée par les hormones chez les vertébrés soit régulée de la même manière chez les invertébrés. Chez les mammifères, par exemple, le neurotransmetteur norépinéphrine augmente la fréquence cardiaque, et chez le crabe Cancer du pagourou et du homard Homarus vulgaris le même rôle est joué par les neurohormones - substances biologiquement actives produites par les cellules neurosécrétoires Tissu nerveux. Le métabolisme du calcium dans le corps est régulé chez les vertébrés par l'hormone des glandes parathyroïdes et chez certains invertébrés - par une hormone produite par un organe spécial situé dans la région thoracique du corps. De nombreuses autres fonctions chez les invertébrés sont également soumises à la régulation hormonale, notamment la métamorphose, le mouvement et le réarrangement des granules pigmentaires dans les chromatophores, la fréquence respiratoire, la maturation des cellules germinales dans les gonades, la formation de caractères sexuels secondaires et la croissance corporelle.

Métamorphose.

Les observations sur les insectes ont révélé le rôle des hormones dans la régulation de la métamorphose, et il a été démontré que plusieurs hormones le faisaient. Nous nous concentrerons sur les deux antagonistes hormonaux les plus importants. À chacun de ces stades de développement accompagnés de métamorphose, les cellules neurosécrétoires du cerveau de l'insecte produisent ce qu'on appelle. hormone cérébrale, qui stimule la synthèse de l'hormone stéroïde qui induit la mue, l'ecdysone, dans la glande prothoracique (prothoracique). Au moment même où l’ecdysone est synthétisée dans le corps de l’insecte, dans les corps adjacents ( corpus allata) - deux petites glandes situées dans la tête d'un insecte - produit ce qu'on appelle. hormone juvénile, qui supprime l'action de l'ecdysone et assure le prochain stade larvaire après la mue. Au fur et à mesure que la larve grandit, de moins en moins d'hormones juvéniles sont produites et, finalement, leur quantité n'est plus suffisante pour empêcher la mue. Par exemple, chez les papillons, une diminution du niveau d'hormone juvénile conduit au fait que le dernier stade larvaire après la mue se transforme en pupe.

L'interaction des hormones régulant la métamorphose a été démontrée dans un certain nombre d'expériences. On sait par exemple que le bug Rhodnius prolixus pendant le cycle de vie normal avant de devenir forme adulte(imago) subit cinq mues. Toutefois, si les larves sont décapitées, les métamorphoses survivantes seront raccourcies et elles développeront des formes adultes, bien que miniatures, mais par ailleurs normales. Le même phénomène peut être observé chez la larve du papillon cécropien du ver à soie ( Samia cécropia), si les corps adjacents sont retirés et éliminent ainsi la synthèse de l'hormone juvénile. Dans ce cas, la même chose que dans Rhodnius, la métamorphose sera raccourcie et les formes adultes seront plus petites que d'habitude. Et vice versa, si les corps adjacents d'une jeune chenille de ver à soie Cecropium sont transplantés dans une larve déjà prête à se transformer en adulte, la métamorphose sera alors retardée et les larves seront plus grosses que d'habitude.

L'hormone juvénile a été récemment synthétisée et peut désormais être obtenue en grande quantité. Des expériences ont montré que si vous influencez l'hormone dans concentrations élevées sur les œufs d'insectes ou à un autre stade de leur développement, lorsque cette hormone est normalement absente, de graves troubles métaboliques surviennent, entraînant la mort de l'insecte. Ce résultat permet d'espérer que l'hormone de synthèse sera nouvelle et très des moyens efficaces antiparasitaire. Par rapport aux insecticides chimiques, l’hormone juvénile présente un certain nombre d’avantages importants. Il n’affecte pas l’activité vitale des autres organismes, contrairement aux pesticides qui perturbent gravement l’écologie de régions entières. Tout aussi important, un insecte peut éventuellement développer une résistance à n’importe quel pesticide, mais il est peu probable qu’un insecte développe une résistance à ses propres hormones.

La reproduction.

Des expériences suggèrent que les hormones sont impliquées dans la reproduction des insectes. Chez les moustiques, par exemple, ils régulent à la fois la formation et la ponte des œufs. Lorsqu'une femelle moustique digère la portion de sang qu'elle a absorbée, les parois de l'estomac et de l'abdomen s'étirent, ce qui déclenche la transmission des impulsions au cerveau. Après environ une heure, des cellules spéciales situées dans la partie supérieure du cerveau libèrent une hormone dans l'hémolymphe (« sang ») circulant dans la cavité corporelle, ce qui stimule la sécrétion d'une autre hormone par deux glandes situées dans la zone de constriction, ou col de l'utérus. Cette deuxième hormone stimule non seulement la maturation des œufs, mais également le stockage des nutriments qu'ils contiennent. Chez les moustiques femelles matures, pendant la journée, sous l'influence de la lumière, une hormone spéciale est libérée dans les centres correspondants du système nerveux qui stimule la ponte des œufs, qui se produit généralement l'après-midi, c'est-à-dire encore pendant la journée. Avec un changement artificiel de la nuit au jour, cet ordre peut être perturbé : dans des expériences avec un moustique Aedes aegypti(porteuse de la fièvre jaune), les femelles pondaient la nuit si elles étaient gardées dans des cages éclairées la nuit et dans des cages sombres pendant la journée. Chez la plupart des espèces d'insectes, la ponte est stimulée par une hormone produite par une certaine partie des corps adjacents.

Chez les blattes, les sauterelles, les punaises de lit et les mouches, la maturation ovarienne dépend d'une des hormones sécrétées par les corps adjacents ; en l’absence de cette hormone, les ovaires ne mûrissent pas. À leur tour, les ovaires produisent des hormones qui affectent les organes adjacents. Ainsi, lorsque les ovaires étaient retirés, une dégénérescence des corps adjacents était observée. Si des ovaires matures étaient transplantés dans un tel insecte, la taille normale des corps adjacents était restaurée après un certain temps.

Différences de sexe.

De nombreux invertébrés, dont les insectes, sont caractérisés par un dimorphisme sexuel, c'est-à-dire différence de caractéristiques morphologiques chez les mâles et les femelles. Chez les moustiques, par exemple, la femelle se nourrit du sang des mammifères et ses pièces buccales sont adaptées pour percer la peau, tandis que les mâles se nourrissent de nectar ou de jus de plantes et ont une trompe plus longue et plus fine. Chez les abeilles, le dimorphisme sexuel est clairement corrélé aux caractéristiques du comportement et du destin de chaque caste d'individus : les mâles (bourdons) ne servent qu'à la reproduction et meurent après le vol nuptial ; les femelles sont représentées par deux castes - la reine (reine), qui a un développement système reproducteur et participe à la reproduction et à la stérilisation des abeilles ouvrières. Les observations et expériences réalisées sur les abeilles et autres invertébrés montrent que le développement des caractères sexuels est régulé par les hormones produites par les gonades.

Chez de nombreux crustacés, l’hormone sexuelle mâle (androgène) est produite par la glande androgène située dans le canal déférent. Cette hormone est nécessaire à la formation des testicules et des organes génitaux accessoires (copulatoires), ainsi qu'au développement des caractères sexuels secondaires. Lorsque la glande androgène est retirée, la forme et la fonction du corps changent, de sorte que l'homme castré finit par ressembler à une femme.

Changement de couleur.

La capacité de changer la couleur du corps est caractéristique de nombreux invertébrés, notamment les insectes, les crustacés et les mollusques. phasme Dixippe sur un fond vert, il apparaît vert, et sur un fond plus foncé, il ressemble à un bâton, comme recouvert d'écorce. Chez les phasmes, comme chez de nombreux autres organismes, le changement de couleur du corps en fonction de la couleur du fond est l'un des principaux moyens de défense, permettant à l'animal d'échapper à l'attention d'un prédateur.

Dans le corps des invertébrés capables de changer la couleur du corps, des hormones sont produites qui stimulent le mouvement et le réarrangement des granules pigmentaires. À la fois dans la lumière et dans temps sombre le jour, le pigment vert est réparti uniformément dans les chromatophores, donc pendant la journée, le phasme est coloré en vert. Dans des conditions de fond éclairé, des granules de pigments bruns et rouges sont regroupés le long des bords de la cellule. Lorsque l’obscurité s’installe ou que la lumière diminue, les granules de pigments foncés se dispersent et l’insecte prend la couleur de l’écorce des arbres. La réponse chromatophore est provoquée par une neurohormone libérée par le cerveau en réponse aux changements d’éclairage de fond. Sous l’influence de la lumière, cette hormone pénètre dans le sang et est délivrée à la cellule cible. D'autres hormones d'insectes qui régulent le mouvement des pigments pénètrent dans le sang à partir des corps adjacents et du ganglion ( ganglion), situé sous l'œsophage.

Les pigments rétiniens de l'œil composé des crustacés se déplacent également en réponse aux changements de lumière, et cette adaptation à la lumière est subordonnée régulation hormonale. Les calmars et autres crustacés possèdent également des cellules pigmentaires dont la réponse à la lumière est régulée par des hormones. Les chromatophores du calmar contiennent des pigments bleus, violets, rouges et jaunes. Avec une stimulation appropriée, son corps peut prendre différentes couleurs, ce qui lui donne la capacité de s'adapter instantanément à son environnement.

Les mécanismes qui contrôlent le mouvement des pigments dans les chromatophores sont différents. A la pieuvre Élédone Les chromatophores contiennent des fibres qui peuvent se contracter sous l'action de la tyramine, une hormone produite par glande salivaire. Lorsqu’ils se contractent, la zone occupée par les pigments s’agrandit et le corps de la pieuvre s’assombrit. Lorsque les fibres se détendent en réponse à une autre hormone, la bétaïne, la zone se contracte et le corps s'éclaircit.

Un mécanisme différent pour le mouvement des pigments a été découvert dans les cellules cutanées des insectes, dans les cellules rétiniennes de certains crustacés et chez les vertébrés à sang froid. Chez ces animaux, les granules de pigments sont associés à des molécules protéiques à haute teneur en polymère capables de passer de l’état sol à l’état gel et vice-versa. Lors du passage à l'état de gel, le volume occupé par les molécules protéiques diminue et des granules pigmentaires s'accumulent au centre de la cellule, ce qui s'observe dans la phase sombre. Dans la phase légère, les molécules de protéines passent à l'état sol ; ceci s'accompagne d'une augmentation de leur volume et d'une dispersion des granules dans toute la cellule.

HORMONES VERTÉBRÉS

Tous les vertébrés ont des hormones identiques ou très similaires, et chez les mammifères, cette similitude est si grande que certains médicaments hormonaux, obtenus à partir d'animaux, sont utilisés pour des injections chez l'homme. Parfois, cependant, une hormone particulière agit différemment selon les espèces. Par exemple, les œstrogènes produits par les ovaires affectent la croissance des plumes chez les poulets Leghorn mais n’affectent pas la croissance des plumes chez les pigeons.

Toutes les études sur le rôle des hormones ne permettent pas de tirer des conclusions claires. Par exemple, les données concernant le rôle des hormones dans les migrations des oiseaux sont contradictoires. Chez certaines espèces, notamment le junco d'hiver, les gonades grossissent au printemps à mesure que la durée du jour augmente, ce qui suggère que les hormones déclenchent la migration. Cependant, cette réaction n’est pas observée chez d’autres espèces d’oiseaux. Le rôle des hormones dans un phénomène tel que l'hibernation chez les mammifères n'est pas non plus clair.

Thyroxine,

l'hormone thyroïdienne des vertébrés, produite par la glande thyroïde, régule le métabolisme basal et les processus de développement. Des expériences ont montré que les reptiles, par exemple, ont des mues périodiques, au moins partiellement régulé par la thyroxine.

Chez les amphibiens, la fonction de la thyroxine a été mieux étudiée chez les grenouilles. Les têtards nourris avec de l'extrait de thyroïde ont cessé de croître et se sont rapidement transformés en petites grenouilles adultes, c'est-à-dire ils ont connu une métamorphose accélérée. Lorsque leur glande thyroïde a été retirée, la métamorphose ne s’est pas produite et ils sont restés des têtards.

La thyroxine joue également un rôle important dans le cycle de vie d’un autre amphibien, le tigre ambystoma. La larve néoténique (capable de se reproduire) d'Ambystoma - axolotl - ne subit généralement pas de métamorphose et reste au stade larvaire. Cependant, si vous ajoutez une petite quantité d'extrait de thyroïde bovine à la nourriture de l'axolotl, une métamorphose se produira et l'axolotl se développera en un petit ambystome noir respirant l'air.

Bilan de l'eau et des ions.

Chez les amphibiens et les mammifères, la diurèse (miction) est stimulée par l'hydrocortisone, une hormone sécrétée par le cortex surrénalien. L'effet inverse - déprimant - sur la diurèse est exercé par une autre hormone, qui est produite par l'hypothalamus, pénètre dans le lobe postérieur de l'hypophyse et, de là, dans la circulation systémique.

Tous les vertébrés, à l'exception des poissons, possèdent des glandes parathyroïdes qui sécrètent une hormone qui aide à maintenir l'équilibre du calcium et du phosphore. Apparemment, chez les poissons osseux, la fonction des glandes parathyroïdes est assurée par d'autres structures, mais cela n'a pas encore été établi avec certitude. D'autres hormones impliquées dans le métabolisme, régulant l'équilibre des ions potassium, sodium et chlore, sont sécrétées par le cortex surrénalien et le lobe postérieur de l'hypophyse. Les hormones du cortex surrénalien augmentent la teneur en ions sodium et chlore dans le sang des mammifères, des reptiles et des grenouilles.

Insuline.

Les deux hormones qui régulent la glycémie, l'insuline et le glucagon, sont produites par des cellules spécialisées du pancréas qui composent les îlots de Langerhans. Il existe quatre types de cellules : alpha, bêta, C et D. La proportion de celles-ci types de cellules varie selon les groupes d'animaux et un certain nombre d'amphibiens n'ont que des cellules bêta. Certaines espèces de poissons n'ont pas de pancréas et le tissu des îlots se trouve dans leur paroi intestinale ; il existe également des espèces chez lesquelles il se trouve dans le foie. Il existe des poissons connus dans lesquels des accumulations de tissus d'îlots se présentent sous la forme de glandes endocrines distinctes. Les hormones sécrétées par les cellules des îlots – l’insuline et le glucagon – remplissent apparemment la même fonction chez tous les vertébrés.

Hormones hypophysaires.

L'hypophyse sécrète diverses hormones ; leur action est bien connue par les observations sur les mammifères, mais ils jouent le même rôle chez tous les autres groupes de vertébrés. Si, par exemple, on injecte à une grenouille femelle en hibernation un extrait de l’hypophyse antérieure, cela stimulera la maturation des œufs et elle commencera à pondre. Chez le tisserand africain, produit par l'hypophyse antérieure hormone gonadotrophine initie la sécrétion de l'hormone sexuelle mâle par les testicules. Cette hormone stimule l'expansion des tubules efférents du testicule, ainsi que la formation de pigment mélanique dans le bec et, par conséquent, l'assombrissement du bec. Chez le même tisserand africain, l'hormone lutéinisante produite par le lobe postérieur de l'hypophyse initie la synthèse de pigments dans certaines plumes et la sécrétion de progestérone par le corps jaune de l'ovaire.

Littérature:

Dogel V.A. Zoologie des invertébrés. M., 1981
Tepperman J., Tepperman H. Physiologie du métabolisme et du système endocrinien. M., 1989
Hadorn E., Wehner. R. Zoologie générale. M., 1989
Alberts B., Bray D., Lewis J., Raff M., Roberts K., Watson J. Biologie cellulaire moléculaire, tome 2. M., 1994
Physiologie humaine, éd. Schmidt R., Tevsa G., vol. 2-3. M., 1996



Les hormones- des produits chimiques de signalisation sécrétés par les glandes endocrines directement dans le sang et ayant un effet complexe et multiforme sur l'organisme dans son ensemble ou sur certains organes et tissus cibles. Les hormones servent de régulateurs humoraux (transmis par le sang) de certains processus dans certains organes et systèmes. Il existe d’autres définitions selon lesquelles l’interprétation du concept d’hormone est plus large : « produits chimiques de signalisation produits par les cellules du corps et affectant les cellules d’autres parties du corps ». Cette définition semble préférable, car elle couvre de nombreuses substances traditionnellement classées comme hormones : hormones d'animaux ne possédant pas de système circulatoire (par exemple, l'ecdysone vers ronds etc.), les hormones des vertébrés qui ne sont pas produites dans les glandes endocrines (prostaglandines, érythropoïétine, etc.), ainsi que les hormones végétales.

Actuellement, plus d'une centaine et demie d'hormones de différentes Organismes multicellulaires. Par structure chimique ils sont divisés en trois groupes : protéine-peptide, dérivés d'acides aminés Et hormones stéroïdes .

Le premier groupe est constitué des hormones de l'hypothalamus et de l'hypophyse, du pancréas et glandes parathyroïdes et la calcitonine, une hormone thyroïdienne. Certaines hormones, telles que les hormones folliculo-stimulantes et thyroïdiennes, sont des glycoprotéines - des chaînes peptidiques « décorées » de glucides.

Dérivés d'acides aminés sont des amines synthétisées dans moelle glandes surrénales (adrénaline et noradrénaline) et dans la glande pinéale (mélatonine), ainsi que les hormones thyroïdiennes contenant de l'iode, la triiodothyronine et la thyroxine (tétraiodothyronine).

Le troisième groupe est précisément responsable de la réputation frivole que les hormones ont acquise auprès du peuple : ce sont les hormones stéroïdes synthétisées dans le cortex surrénalien et dans les gonades. En regardant leur formule générale, il est facile de deviner que leur précurseur biosynthétique est le cholestérol. Les stéroïdes diffèrent par le nombre d'atomes de carbone dans la molécule : C21 - hormones surrénales et progestérone, C19 - hormones sexuelles mâles (androgènes et testostérone), C18 - hormones sexuelles féminines (œstrogènes).

Les molécules d'hormones hydrophiles, par exemple les molécules protéines-peptides, sont généralement transportées dans le sang sous forme libre, et les hormones stéroïdes ou les hormones thyroïdiennes contenant de l'iode sont transportées sous forme de complexes avec les protéines du plasma sanguin. D'ailleurs, les complexes protéiques peuvent également servir de réserve d'hormone ; lorsque la forme libre de l'hormone est détruite, le complexe avec la protéine se dissocie et est ainsi maintenu. concentration requise molécule de signalisation.

Après avoir atteint la cible, l'hormone se lie au récepteur - une molécule protéique dont une partie est responsable de la liaison et de la réception du signal, l'autre de la transmission de l'effet « course de relais » dans la cellule. (En règle générale, l'activité de certaines enzymes change.) Les récepteurs des hormones hydrophiles sont situés sur les membranes des cellules cibles et les récepteurs des hormones lipophiles sont situés à l'intérieur des cellules, car les molécules lipophiles peuvent pénétrer dans la membrane. Les signaux des récepteurs sont reçus par ce que l'on appelle des seconds messagers, ou intermédiaires, qui sont beaucoup moins diversifiés que les hormones elles-mêmes. Ici, nous rencontrons des personnages aussi familiers que le cyclo-AMP, les protéines G, les protéines kinases - des enzymes qui attachent des groupes phosphate aux protéines, générant ainsi de nouveaux signaux. Maintenant, relevons-nous de niveau cellulaire au niveau des organes et des tissus. De ce point de vue, tout commence dans l’hypothalamus et l’hypophyse. Les fonctions de l'hypothalamus sont diverses et ne sont pas encore entièrement comprises aujourd'hui, mais tout le monde s'accorde probablement sur le fait que le complexe hypothalamo-hypophysaire est le point central d'interaction entre les systèmes nerveux et endocrinien. L'hypothalamus est également le centre de régulation fonctions végétatives, et « le berceau des émotions ». Il produit des hormones de libération (de l'anglais release - to release), ce sont aussi des libérines, qui stimulent la libération d'hormones par l'hypophyse, ainsi que des statines, qui inhibent cette libération.

Pituitaire- un organe endocrinien situé à la surface interne du cerveau. Il produit des hormones tropiques (du grec tropos - direction), ainsi appelées parce qu'elles dirigent le travail d'autres glandes endocrines périphériques - les glandes surrénales, les glandes thyroïde et parathyroïde, le pancréas et les gonades. De plus, ce schéma est saturé retour, par exemple, l'hormone féminine estradiol, entrant dans l'hypophyse, régule la sécrétion d'hormones triples qui contrôlent sa propre sécrétion. Par conséquent, la quantité d’hormone, d’une part, n’est jamais excessive et, d’autre part, divers processus endocriniens sont finement coordonnés les uns avec les autres. Attention particulière mérite une réglementation temporaire. L'« horloge intégrée » de notre corps est la glande pinéale, glande pinéale, qui produit l'hormone mélatonine (un dérivé de l'acide aminé tryptophane). Les changements dans la concentration de cette substance créent la notion du temps chez une personne, et la nature de ces changements détermine si une personne sera un « oiseau de nuit » ou une « alouette ». La concentration de nombreuses hormones change également de manière cyclique tout au long de la journée. C'est pourquoi les endocrinologues exigent parfois que les patients collectent quotidiennement l'urine (la quantité peut s'avérer être une valeur plus constante et caractéristique que les termes), et parfois, s'ils ont besoin d'évaluer la dynamique, ils passent des tests toutes les heures.

Hormone somatotrope(GH) a un effet sur tout le corps : elle stimule la croissance et régule en conséquence les processus métaboliques.

Les tumeurs hypophysaires qui provoquent une surproduction de cette hormone provoquent le gigantisme chez l'homme et l'animal. Si la tumeur ne survient pas pendant l'enfance, mais plus tard, une acromégalie se développe - une croissance inégale du squelette, principalement due aux zones cartilagineuses. Au contraire, une insuffisance en hormone de croissance conduit au nanisme, ou nanisme hypophysaire. Heureusement, la médecine moderne guérit ce problème. Si le médecin détermine que la cause est trop croissance lente enfant (pas même nécessairement de nanisme, mais simplement en retard sur ses pairs) précisément dans une faible concentration de GH, et juge nécessaire de lui prescrire des injections d'hormones, alors la croissance est normalisée. Mais l'histoire de l'écrivain de science-fiction soviétique Alexandre Belyaev « L'homme qui a trouvé son visage » est toujours un conte de fées : les injections hormonales n'aideront pas un adulte à grandir.

L'hypophyse produit également de la prolactine, également connue sous le nom d'hormone lactogène et lutéotrope (LTH), responsable de la lactation pendant l'allaitement. De plus, l'hypophyse synthétise les lipotropines - des hormones qui stimulent l'implication des graisses dans le métabolisme énergétique. Ces mêmes hormones sont les précurseurs des endorphines – les « peptides de la joie ».

Les hormones mélanocytaires de l'hypophyse (MSH) régulent la synthèse des pigments dans la peau et, en outre, selon certaines données, elles auraient quelque chose à voir avec les mécanismes de mémoire. Deux hormones plus importantes sont la vasopressine et l’ocytocine ; la première est aussi appelée hormone antidiurétique, elle régule le métabolisme eau-sel et le tonus des artérioles ; L'ocytocine est responsable de l'activité contractile de l'utérus chez les mammifères et, avec la prolactine, du lait. Il est utilisé pour déclencher le travail. Maintenant plus sur hormones tropiques, qui sont produits par l’hypophyse, et sur leurs cibles.

Glandes surrénales- des organes appariés adjacents aux sommets des reins. Chacune d'elles contient deux glandes indépendantes : le cortex (substantia corticalis) et la moelle. La cible de l’hormone adrénocorticotrope (ACTH, également connue sous le nom de corticotropine) est le cortex surrénalien. Les corticostéroïdes sont synthétisés ici. Les glucocorticoïdes (cortisol et autres) tirent leur nom du glucose car leur activité est étroitement liée au métabolisme des glucides.

Le cortisol est une hormone du stress, il protège l'organisme de tout changements soudainséquilibre physiologique : affecte le métabolisme des glucides, des protéines et des lipides, l'équilibre électrolytique. Cependant, ces derniers sont davantage liés aux minéralocorticoïdes : leur principal représentant, l'aldostérone, régule les échanges d'ions sodium, potassium et hydrogène. Les corticostéroïdes et leurs analogues artificiels sont largement utilisés en médecine. Les glucocorticoïdes ont encore une chose propriété importante: ils suppriment les réactions inflammatoires et réduisent la formation d'anticorps, c'est pourquoi des onguents sont fabriqués à partir d'eux pour traiter l'inflammation cutanée et les démangeaisons. À propos, certains sont populaires parmi les amateurs de médecine alternative onguents pour la peau d'origine chinoise, en plus des extraits de plantes, contiennent les mêmes glucocorticoïdes. Ceci est écrit en clair sur l’emballage, mais les acheteurs ne font pas toujours attention aux mots biochimiques complexes. Bien que, peut-être, pour le traitement de la dermatite, il serait préférable d'acheter un fluorocort banal, au moins il est approuvé par la pharmacopée russe...

Les catécholamines - adrénaline et noradrénaline - sont synthétisées dans la médullosurrénale. Aujourd’hui tout le monde sait que l’adrénaline est synonyme de stress. Il est responsable de la mobilisation des réactions adaptatives : il agit à la fois sur le métabolisme et système cardiovasculaire, et sur le métabolisme des glucides et des graisses. Les catécholamines sont la structure la plus simple et, évidemment, les substances de signalisation les plus anciennes ; ce n'est pas pour rien qu'on les trouve même chez les protozoaires. Mais ils ne jouent un rôle particulier de neurotransmetteur que dans les organismes multicellulaires. Nous en reparlerons une autre fois.

Pancréas- simultanément exocrine et endocrine, c'est-à-dire qu'il agit à la fois en externe et en interne : il sécrète des enzymes dans le duodénum (contenu tube digestif les biologistes le considèrent comme un environnement extérieur au corps) et les hormones - dans le sang.

Dans les formations glandulaires spéciales, les îlots de Langerhans, les cellules alpha produisent du glucagon, un régulateur du métabolisme des glucides et des graisses, et les cellules bêta produisent de l'insuline. Cette hormone a été découverte par le scientifique russe L.V. Sobolev (1902). L'insuline a été isolée pour la première fois par les physiologistes canadiens Frederick Banting, Charles Best et John MacLeod (1921). Banting et MacLeod reçurent pour cela le prix Nobel en 1923. (Best, qui occupait le poste d'assistant de laboratoire, n'était pas inclus dans le nombre de lauréats, et Banting, indigné, a remis la moitié de sa récompense à l'assistant.)

L'unité structurelle de l'insuline est un monomère d'un poids moléculaire d'environ 6 000, et de deux à six monomères sont combinés en une molécule. La séquence d'arrangement des acides aminés dans le monomère d'insuline (c'est-à-dire sa structure primaire) a été établie pour la première fois par le biochimiste anglais Frederick Sanger (1956, prix Nobel de chimie 1958), et la structure spatiale a de nouveau été établie par l'Anglaise et également Dorothy Hodgkin, lauréate du prix Nobel (1972). Chaque monomère contient 51 acides aminés, disposés sous la forme de deux chaînes peptidiques - A et B, reliées par deux ponts disulfure (-S-S-).

Insuline. Cette hormone abaisse la glycémie en retardant la dégradation du glycogène et la synthèse du glucose dans le foie et en augmentant en même temps la perméabilité des membranes cellulaires au glucose. Il favorise également l'absorption de ce carburant, stimule la synthèse des protéines et des graisses au détriment des glucides. Ainsi, il est chargé de garantir que les cellules absorbent le glucose du sang et le « digèrent » bien.

Le manque d’insuline signifie une augmentation de la glycémie et une « faim » de cellules, de tissus et d’organes, en d’autres termes, un diabète sucré. C'est probablement le plus célèbre maladie endocrinienne. En particulier parce que l'insuline est la première hormone peptidique synthétisée artificiellement, qui a remplacé les médicaments obtenus à partir du pancréas des bovins de boucherie. Aujourd’hui, les médecins rêvent de succès encore plus radicaux, par exemple en introduisant des cellules souches qui produisent de l’insuline dans le corps d’un patient. L'introduction d'une telle technique dans pratique clinique- ce n'est pas une tâche facile ni rapide, mais les injections d'insuline permettent vie normaleà beaucoup de gens aujourd'hui.

Hormone stimulant la thyroïde de l'hypophyse(TSH) agit sur la glande thyroïde (glandula thyroïdea), qui chez nous, être humain, est située dans le cou, sous le larynx. Ses hormones sont la thyroxine et la triiodothyronine, régulatrices du métabolisme, de la synthèse des protéines, de la différenciation tissulaire, du développement et de la croissance de l'organisme. Leur précurseur biochimique est l’acide aminé tyrosine. Étant donné que les molécules d’hormones thyroïdiennes contiennent de l’iode, une carence en cet élément dans les aliments entraîne une carence hormonale.

Les manifestations cliniques sont la croissance de la glande (goitre) avec une diminution de sa fonction. Goitre toxique, également connue sous le nom de maladie de Basedow, ou thyréotoxicose, au contraire, est associée à un hyperfonctionnement de la glande et à des taux hormonaux excessifs. La glande thyroïde synthétise également une hormone qui régule le métabolisme du calcium et du phosphore, la calcitonine. Et une autre hormone qui régule l'échange de ces mêmes éléments est produite par des glandes parathyroïdes appariées - elle s'appelle l'hormone parathyroïdienne. Ces hormones, ainsi que la vitamine D, sont responsables de la croissance et de la réparation du tissu osseux.

Hormones gonadotropes de l'hypophyse- hormone lutéinisante (LH), gonadotrophine, hormone folliculo-stimulante Hormone FSH réguler l'activité des glandes sexuelles. (Nous y sommes finalement parvenus.) La testostérone, le principal androgène, est produite par les testicules chez l'homme, et par le cortex surrénalien et les ovaires chez la femme. Sur la scène développement intra-utérin Cette hormone chez l'homme dirige la différenciation des organes génitaux et, pendant la puberté, le développement des caractères sexuels secondaires, ainsi que la formation de l'orientation sexuelle masculine.

Chez l'adulte, la testostérone apporte fonctionnement normal organes génitaux. À propos, les testicules de l’embryon d’un garçon produisent également le facteur de régression du canal Müllérien, une hormone qui bloque le développement du système reproducteur féminin. Ainsi, au cours de la période embryonnaire, le développement d'un garçon s'accompagne de signaux chimiques que les filles n'ont pas, et c'est là que surgissent finalement toutes les autres différences. Comme le disent les experts en plaisantant, « pour avoir un garçon, il faut faire quelque chose, mais si vous ne faites rien, vous aurez une fille ». Les œstrogènes chez les femmes sont synthétisés dans les ovaires. L'estradiol, l'un des principaux œstrogènes, est responsable de la formation des caractères sexuels féminins secondaires et participe à la régulation du cycle mensuel.

Progestatifs(progestérone et ses dérivés) sont nécessaires à la fois à la régulation du cycle et au déroulement normal de la grossesse. Sans fécondation, les cellules synthétisent de la progestérone à une certaine période du cycle et au cours des 12 premières semaines corps jaune les ovaires, puis le placenta. La progestérone est également sécrétée dans petites quantités le cortex surrénalien et, chez l'homme, les testicules. Généralement, la progestérone est un intermédiaire dans la synthèse des androgènes.

Les ovaires synthétisent également de la relaxine, une hormone de naissance chargée par exemple de détendre les ligaments pelviens. Mais peut-être qu'aucune substance contenue dans le corps humain n'évoque autant d'émotions chez le beau sexe que la gonadotrophine chorionique humaine. Le placenta fœtal peut également être considéré comme un organe endocrinien : il synthétise le progestatif, la relaxine et de nombreuses autres hormones et substances analogues aux hormones. Futur enfantéchange constamment des signaux avec le corps de la mère, créant ainsi des conditions appropriées pour lui-même. L’une des premières tentatives du fœtus pour établir un lien avec sa mère est précisément cette glycoprotéine, la gonadotrophine chorionique humaine, également connue sous le nom de HGT ou hCG. Sa présence dans le sang ou l’urine d’une femme signifie que la patiente est enceinte, et son absence signifie que la grossesse, hélas (ou hourra), n’a pas eu lieu. Au milieu du siècle dernier, cette analyse fatidique était complètement barbare : on injectait de l’urine de femme à des souris et on regardait si les animaux présentaient des symptômes de grossesse. Aujourd'hui, il se distingue par son élégante simplicité : vous n'avez même pas besoin d'aller chez le médecin, il suffit d'acheter un test de grossesse à la pharmacie, également connu sous le nom de « bandelette » - une bandelette étroite dans une enveloppe, essentiellement une pièce chromatographique miniature. de papier.

Il est difficile de trouver un autre exemple où l'amélioration d'une technique de routine analyse biochimique aurait un impact si profond sur les destinées humaines. Combien de grossesses préservées et combien d'avortements en temps opportun... Eh bien, oui, sans aucun doute, l'avortement est mauvais. Mais il n’appartient pas à la médecine de faire en sorte que les gens ne commettent pas de bêtises. Avec cela - aux psychologues, aux enseignants et aux économistes. Les médecins et les scientifiques ne peuvent que minimiser les dommages causés par la stupidité.

Mécanismes d'action des hormones Lorsqu’une hormone présente dans le sang atteint une cellule cible, elle interagit avec des récepteurs spécifiques ; les récepteurs « lisent le message » du corps et certains changements commencent à se produire dans la cellule. Chaque hormone spécifique correspond exclusivement à « ses propres » récepteurs situés dans des organes et des tissus spécifiques - ce n'est que lorsque l'hormone interagit avec eux qu'un complexe hormone-récepteur se forme.

Les mécanismes d'action des hormones peuvent être différents. L'un des groupes est constitué d'hormones qui se lient aux récepteurs situés à l'intérieur des cellules, généralement dans le cytoplasme. Il s'agit notamment des hormones ayant des propriétés lipophiles - par exemple, les hormones stéroïdes (sexuelles, gluco- et minéralocorticoïdes), ainsi que les hormones thyroïdiennes. Étant liposolubles, ces hormones pénètrent facilement dans la membrane cellulaire et commencent à interagir avec les récepteurs du cytoplasme ou du noyau. Ils sont légèrement solubles dans l’eau et, lorsqu’ils sont transportés dans le sang, ils se lient aux protéines porteuses. On pense que dans ce groupe d'hormones, le complexe hormone-récepteur agit comme une sorte de relais intracellulaire - s'étant formé dans la cellule, il commence à interagir avec la chromatine, qui est située dans les noyaux cellulaires et est constituée d'ADN et de protéines, et ainsi accélère ou ralentit le travail de certains gènes. En influençant sélectivement un gène spécifique, l'hormone modifie la concentration de l'ARN et de la protéine correspondants, tout en corrigeant les processus métaboliques.

Le résultat biologique de chaque hormone est très spécifique. Bien que les hormones modifient généralement moins de 1 % des protéines et de l'ARN de la cellule cible, cela est tout à fait suffisant pour obtenir l'effet physiologique correspondant. La plupart des autres hormones se caractérisent par trois caractéristiques :

  • ils se dissolvent dans l'eau ;
  • ne se lient pas aux protéines porteuses ;
  • commencent le processus hormonal dès qu'ils se connectent au récepteur, qui peut être situé dans le noyau cellulaire, son cytoplasme ou situé à la surface de la membrane plasmique.

Le mécanisme d'action du complexe hormone-récepteur de ces hormones implique nécessairement des intermédiaires qui induisent une réponse cellulaire. Les plus importants de ces médiateurs sont l’AMPc (adénosine monophosphate cyclique), l’inositol triphosphate et les ions calcium. Ainsi, dans un environnement dépourvu d’ions calcium, ou dans des cellules qui n’en contiennent pas suffisamment, l’effet de nombreuses hormones est affaibli ; lors de l'utilisation de substances qui augmentent la concentration intracellulaire de calcium, des effets identiques à ceux de certaines hormones se produisent.

La participation des ions calcium comme intermédiaire assure l'effet sur les cellules d'hormones telles que la vasopressine et les catécholamines. Il existe cependant des hormones pour lesquelles aucun messager intracellulaire n’a encore été découvert. Parmi les plus connues de ces hormones se trouve l’insuline, dans laquelle il a été proposé que l’AMPc et le GMPc jouent le rôle de médiateur, ainsi que les ions calcium et même le peroxyde d’hydrogène, mais il n’existe toujours aucune preuve convaincante en faveur d’une substance particulière. De nombreux chercheurs pensent que dans ce cas, les médiateurs peuvent être des composés chimiques dont la structure est complètement différente de celle des médiateurs déjà connus de la science. Une fois leur tâche accomplie, les hormones sont soit décomposées dans les cellules cibles ou dans le sang, soit transportées vers le foie où elles sont décomposées, soit finalement éliminées de l'organisme principalement par l'urine (par exemple, l'adrénaline).

Introduction

    Concept du système endocrinien

    Le concept des hormones et leur importance dans le corps

    Le concept de cellules cibles et de récepteurs hormonaux

  1. Cellules uniques productrices d'hormones d'organes non endocriniens

    Conclusion

    Littérature

Introduction

Le système endocrinien a si soigneusement gardé ses secrets qu'il n'a été découvert par les scientifiques qu'au début du XXe siècle. Certes, un peu plus tôt, des chercheurs ont attiré l'attention sur d'étranges incohérences dans la structure de certains organes. En apparence, ces formations anatomiques ressemblaient à des glandes, ce qui signifie qu'elles devaient sécréter certains fluides (« jus » ou « secrets »), tout comme les glandes salivaires produisent de la salive, les glandes lacrymales produisent des larmes, etc.

Le système endocrinien est un système de glandes qui produisent des hormones et les libèrent directement dans le sang. Ces glandes, appelées glandes endocrines ou endocrines, ne possèdent pas de canaux excréteurs ; ils sont situés dans différentes parties du corps, mais sont fonctionnellement étroitement liés. Le système endocrinien du corps dans son ensemble maintient la constance de l'environnement interne nécessaire au déroulement normal des processus physiologiques.

Les hormones sont sécrétées à des rythmes différents, en fonction de la concentration de certains substrats, ions et neurotransmetteurs dans le sang. La sécrétion de chaque hormone se produit sous l'influence d'un signal correspondant. Les hormones stéroïdes et peptidiques libérées dans le sang se lient à des protéines spéciales et sont transportées par le sang à l'état inactif. Une propriété commune des hormones est la dépendance de l'efficacité de la réponse à celles-ci sur la concentration de la fraction libre et la sensibilité des récepteurs à celles-ci.

Concept du système endocrinien

Le système endocrinien comprend un certain nombre de glandes et de cellules individuelles du corps, dont la caractéristique commune et distinctive est la production de substances biologiquement actives - les hormones. Ces derniers sont des intermédiaires dans la régulation des fonctions des organes et de leurs systèmes. Il existe plusieurs classes d'hormones : les peptides (oligopeptides, polypeptides, glycopeptides), les dérivés d'acides aminés (neuroamines) et les stéroïdes (hormones sexuelles, corticostéroïdes). Toutes ces substances biologiquement actives sont produites en très petites quantités.

Une fois dans le sang ou la lymphe, ils entrent en connexion spécifique avec des récepteurs situés à la surface des cellules des organes cibles. Dans ce cas, l'action à distance des organes du système endocrinien sur le corps est réalisée. En plus de la sécrétion endocrinienne elle-même, dans laquelle les hormones sont libérées dans le sang ou la lymphe, il existe également la sécrétion paracrine, lorsque l'hormone se lie à des cellules cibles directement adjacentes aux cellules endocrines, ainsi que la sécrétion autocrine, dans laquelle une hormone qui est sécrété dans une partie de la cellule se lie aux récepteurs d’une autre zone.

Le mécanisme d’action des hormones peut être décrit comme suit. Une molécule hormonale qui circule dans le flux sanguin ou lymphatique « trouve » son récepteur à la surface de la membrane plasmique, dans le cytoplasme ou le noyau d’une cellule cible particulière. Le rôle déterminant dans cette reconnaissance très spécifique est joué par la correspondance stéréochimique entre le centre actif de la molécule hormonale et la configuration de son récepteur. La liaison d'une hormone à un récepteur provoque des changements conformationnels (volumes-spatiaux) dans la molécule réceptrice, qui, à leur tour, affectent systèmes enzymatiques cellules, en particulier sur le système adénylate cyclase. Le mécanisme d'action des hormones est discuté plus en détail dans les manuels de biochimie et de physiologie. L'effet des hormones peut se manifester non seulement en améliorant, mais également en inhibant l'activité des cellules et de leurs systèmes.

Classiquement, parmi les éléments du système endocrinien de l'organisme, on distingue quatre groupes de composants. Le premier groupe - les organes centraux du système endocrinien - comprend l'hypothalamus, l'hypophyse et la glande pinéale. Ces organes sont étroitement liés aux organes du système nerveux central et coordonnent les activités de toutes les autres parties du système endocrinien. Le deuxième groupe est périphérique organes endocriniens- comprend la thyroïde, les glandes thyroïdes et les glandes surrénales.

Le concept des hormones et leur importance dans le corps

Les hormones sont des substances biologiquement actives sécrétées par des glandes endocrines spéciales en réponse à des stimuli spécifiques, qui sont sécrétées dans le sang et délivrées à des tissus cibles qui possèdent des molécules protéiques spécifiques-récepteurs pour une hormone donnée, et les récepteurs transmettent un signal du messager principal ou hormone dans la cellule.

Les hormones sont des composés organiques produits par certaines cellules et conçus pour contrôler, réguler et coordonner les fonctions du corps. Les animaux supérieurs disposent de deux systèmes de régulation à l'aide desquels le corps s'adapte aux changements internes et externes constants. L'un d'eux est le système nerveux, qui transmet rapidement des signaux (sous forme d'impulsions) à travers un réseau de nerfs et de cellules nerveuses ; l'autre est endocrinien, qui effectue une régulation chimique à l'aide d'hormones transportées dans le sang et ayant un effet sur les tissus et organes éloignés du lieu de leur libération. Tous les mammifères, y compris les humains, ont des hormones ; on les trouve également dans d'autres organismes vivants.

Les hormones régulent l’activité de toutes les cellules du corps. Ils affectent l'acuité mentale et la mobilité physique, le physique et la taille, déterminent la croissance des cheveux, le ton de la voix, la libido et le comportement. Grâce au système endocrinien, une personne peut s'adapter à de fortes fluctuations de température, à un excès ou à un manque de nourriture et au stress physique et émotionnel.

L'étude de l'action physiologique des glandes endocrines a permis de révéler les secrets de la fonction sexuelle et le miracle de l'accouchement, ainsi que de répondre à la question de savoir pourquoi certaines personnes sont grandes et d'autres petites, certaines sont grosses, d'autres sont minces. , certains sont lents, d'autres sont agiles, certains sont forts, d'autres sont faibles.

A l'état normal, il existe un équilibre harmonieux entre l'activité des glandes endocrines, l'état du système nerveux et la réponse des tissus cibles (tissus ciblés). Toute violation de chacun de ces liens entraîne rapidement des écarts par rapport à la norme.

Fondamentalement, le rôle des hormones se résume à affiner le corps pour qu’il fonctionne correctement. A titre d'exemple, prenons une hormone antidiurétique (c'est-à-dire antidiurétique), qui est responsable de la régulation de l'excrétion d'eau par les reins. Tout d’abord, cette hormone élimine du sang, ainsi que d’autres déchets, de grandes quantités d’eau dont le corps n’a plus besoin. Cependant, si tout sortait du corps avec l'urine, le corps perdrait trop d'eau et pour éviter que cela ne se produise, une autre partie du rein absorbe à nouveau autant d'humidité que dans l'urine. ce moment votre corps a besoin.

La régulation du système hormonal humain est un processus très délicat. Les glandes productrices d’hormones interagissent étroitement entre elles, ainsi qu’avec le système nerveux du corps. L'importance des hormones pour le maintien de la vie et de la santé humaine est énorme. Le mot « hormone » lui-même vient d’un mot grec qui peut être grossièrement traduit par « boost ». Ce nom indique indirectement que les hormones agissent comme catalyseurs des changements chimiques au niveau cellulaire nécessaires à la croissance, au développement et à la production d'énergie.

Les hormones, une fois dans la circulation sanguine, doivent se déplacer vers les organes cibles appropriés. Le transport des hormones (protéines) de haut poids moléculaire a été peu étudié en raison du manque de données précises sur le poids moléculaire et la structure chimique de bon nombre d'entre elles. Les hormones ayant un poids moléculaire relativement faible se lient rapidement aux protéines plasmatiques, de sorte que la teneur en hormones dans le sang sous forme liée est plus élevée que sous forme libre ; ces deux formes sont en équilibre dynamique. Ce sont les hormones libres qui présentent une activité biologique et, dans de nombreux cas, il a été clairement démontré qu'elles sont extraites du sang par les organes cibles. L’importance de la liaison des hormones aux protéines dans le sang n’est pas entièrement claire.

Pour que le principal type de carburant des cellules - le glucose - pénètre dans le sang, il est nécessaire de le libérer des principaux sites de stockage. Plusieurs hormones agissent comme des « cambrioleurs » dans le corps. Lorsque les muscles nécessitent une injection urgente d’énergie, le corps commence à libérer du glucagon, une hormone produite par des cellules spéciales du pancréas. Cette hormone aide à libérer le glucose dans le sang, qui est stocké dans le foie sous forme de glycogène glucidique.

Pour qu’une cellule du corps puisse utiliser efficacement le glucose, l’hormone insuline, produite dans le pancréas, est nécessaire. Il régule le taux de consommation de glucose dans le corps et le manque d'insuline entraîne une maladie grave : le diabète. La somatotropine produite dans l'hypophyse est responsable de la croissance corporelle. La testostérone régule la croissance des tissus musculaires et osseux, ainsi que la croissance de la barbe. Cette hormone dirige l’énergie et les matériaux pour créer une masse musculaire supplémentaire. Par conséquent, grâce à sa plus grande quantité que les femmes, les hommes perdent du poids plus rapidement.

Le concept de cellules cibles et de récepteurs hormonaux

Les cellules cibles sont des cellules qui interagissent spécifiquement avec les hormones à l'aide de protéines réceptrices spéciales. Ces protéines réceptrices sont situées sur la membrane externe de la cellule, ou dans le cytoplasme, ou sur la membrane nucléaire et d'autres organites de la cellule.

Chaque cellule cible possède un récepteur spécifique pour l’action de l’hormone, et certains de ces récepteurs sont situés dans la membrane. Ce récepteur est stéréospécifique. Dans d'autres cellules, les récepteurs sont situés dans le cytoplasme. Ce sont des récepteurs cytosoliques qui réagissent avec l'hormone qui pénètre dans la cellule. Par conséquent, les récepteurs sont divisés en récepteurs membranaires et cytosoliques. Pour qu’une cellule réponde à l’action d’une hormone, la formation de messagers secondaires à l’action des hormones est nécessaire. Ceci est typique des hormones avec une réception de type membranaire.

La destruction de l'AMP cyclique se produit sous l'action de l'enzyme phosphodiestérase. Le GMF cyclique a l’effet inverse. Lorsque la phospholipase C est activée, des substances se forment qui favorisent l'accumulation de calcium ionisé à l'intérieur de la cellule. Le calcium active les protéines cinases et favorise la contraction musculaire. Le diacylglycérol favorise la conversion des phospholipides membranaires en acide arachidonique, source de formation de prostaglandines et de leucotriènes.

La plupart des récepteurs n'ont pas été suffisamment étudiés car leur isolement et leur purification sont très difficiles et la teneur de chaque type de récepteur dans les cellules est très faible. Mais on sait que les hormones interagissent avec leurs récepteurs par des moyens physiques et chimiques. Des interactions électrostatiques et hydrophobes se forment entre la molécule hormonale et le récepteur. Lorsque le récepteur se lie à une hormone, des changements conformationnels se produisent dans la protéine réceptrice et le complexe de la molécule de signalisation avec la protéine réceptrice est activé. Dans son état actif, il peut provoquer des réactions intracellulaires spécifiques en réponse à un signal reçu.

Selon la structure de l’hormone, il existe deux types d’interactions. Si la molécule hormonale est lipophile (par exemple, les hormones stéroïdes), elle peut alors pénétrer dans la couche lipidique de la membrane externe des cellules cibles. Si une molécule a grandes tailles ou est polaire, alors sa pénétration dans la cellule est impossible. Par conséquent, pour les hormones lipophiles, les récepteurs sont situés à l’intérieur des cellules cibles, et pour les hormones hydrophiles, les récepteurs sont situés dans la membrane externe.

Pour obtenir une réponse cellulaire à un signal hormonal dans le cas de molécules hydrophiles, un mécanisme de transduction de signal intracellulaire intervient. Cela se produit avec la participation de substances appelées seconds messagers. Les molécules hormonales ont des formes très diverses, mais pas les « seconds messagers ».

Il existe deux manières principales de transmettre un signal aux cellules cibles à partir de molécules de signalisation avec mécanisme à membrane Actions:

    les systèmes d'adénylate cyclase (ou guanylate cyclase) ;

    mécanisme phosphoinositide.

Les mécanismes de transmission des informations provenant des hormones à l'intérieur des cellules cibles à l'aide des intermédiaires répertoriés présentent des caractéristiques communes :

    l'une des étapes de la transmission du signal est la phosphorylation des protéines ;

    la cessation de l'activation se produit à la suite de mécanismes spéciaux initiés par les participants au processus eux-mêmes - il existe des mécanismes de rétroaction négative.

Les hormones sont les principaux régulateurs humoraux des fonctions physiologiques de l'organisme et leurs propriétés, processus de biosynthèse et mécanismes d'action sont désormais bien connus.

Glande pinéale

Glande pinéale, une petite formation située chez les vertébrés sous le cuir chevelu ou profondément dans le cerveau ; situé sur la ligne médiane du corps, comme le cœur, il fonctionne soit comme un organe de perception de la lumière, soit comme une glande endocrine dont l'activité dépend de l'éclairage. Il se forme lors de l'embryogenèse sous la forme d'une petite saillie de la paroi dorsale de la vésicule médullaire intermédiaire. Il produit et libère dans le sang des hormones qui régulent tous les changements cycliques du corps : quotidiens, rythmes circadiens. Il reçoit une stimulation lumineuse de la rétine via les voies nerveuses sympathiques, les cycles mensuels. Chez certaines espèces de vertébrés, les deux fonctions sont combinées. Chez l'homme, cette formation a la forme d'une pomme de pin, d'où son nom (du grec épiphyse - cône, croissance).

L'épiphyse est recouverte à l'extérieur d'une capsule de tissu conjonctif, à partir de laquelle s'étendent de minces septa de tissu conjonctif, divisant la glande en lobules indistincts. Les septa contiennent des hémocapillaires. Le stroma des lobules est constitué de cellules gliales, leur concentration augmente vers la périphérie, où elles forment un voile marginal, et les pinéalocytes sont situés au centre. Ce sont des cellules neurosécrétoires, elles ont un gros noyau, des organites bien développés et les processus de ces cellules s'étendent dans les septa du tissu conjonctif et se terminent au niveau des hémocapillaires. Ces cellules produisent la neuroamine sérotonine. Il est produit pendant la journée et la nuit, il est converti en hormone sérotonine. Ces hormones agissent sur l'hypothalamus.

La sérotonine améliore la fonction et la mélatonine l’affaiblit. Ces hormones inhibent le développement du système reproducteur. La glande pinéale produit de l'hormone antigonadotrope ; une hormone qui régule le métabolisme minéral ; un grand nombre de peptides régulateurs (libérines et statines), qui exercent leurs effets soit par l'intermédiaire de l'hypothalamus, soit directement sur l'hypophyse. La glande pinéale atteint son développement maximum vers l'âge de 5-7 ans, puis elle s'atrophie et sa minéralisation se produit (des sels de Ca se déposent).

La glande pinéale se développe au cours de l'embryogenèse à partir du fornix (épithalamus) de la partie postérieure (diencéphale) du cerveau antérieur. Les vertébrés inférieurs, comme les lamproies, peuvent développer deux structures similaires. L'un, situé avec côté droit cerveau, s’appelle la glande pinéale, et la seconde, à gauche, est la glande parapinéale. La glande pinéale est présente chez tous les vertébrés, à l'exception des crocodiles et de certains mammifères, comme les fourmiliers et les tatous. La glande parapinéale en tant que structure mature n'est présente que chez certains groupes de vertébrés, comme les lamproies, les lézards et les grenouilles. .

Cellules uniques productrices d'hormones d'organes non endocriniens

Un ensemble de cellules productrices d’hormones uniques est appelé système endocrinien diffus. Un nombre important de ces endocrinocytes se trouvent dans les muqueuses de divers organes et dans les glandes qui leur sont associées. Ils sont particulièrement nombreux dans les organes du système digestif.

Les cellules du système endocrinien diffus dans les muqueuses ont une base large et une partie apicale plus étroite. Dans la plupart des cas, ils sont caractérisés par la présence de granules sécrétoires denses argyrophiles dans coupes basales cytoplasme. Les produits de sécrétion des cellules du système endocrinien diffus ont des influences endocriniennes à la fois locales (paracrines) et distantes. Les effets de ces substances sont très variés.

Parmi les cellules productrices d'hormones uniques, on distingue deux groupes indépendants : I - les cellules neuroendocrines de la série APUD (origine nerveuse) ; II - cellules d'origine non nerveuse.

Le premier groupe comprend les neurocytes sécrétoires, formés de neuroblastes, qui ont la capacité de produire simultanément des neuroamines et également de synthétiser des hormones protéiques, c'est-à-dire ayant les caractéristiques des cellules nerveuses et endocrines, donc appelées cellules neuroendocrines.

Le deuxième groupe comprend les cellules des organes endocriniens et non endocriniens qui sécrètent des stéroïdes et d'autres hormones : l'insuline (cellules B), le glucagon (cellules A), les peptides (cellules D, cellules K), la sécrétine (cellules S) . Il s'agit également des cellules de Leydig (glandulocytes) du testicule, qui produisent de la testostérone, et des cellules de la couche granulaire des follicules ovariens, qui produisent des œstrogènes et de la progestérone, qui sont des hormones stéroïdes. La production de ces hormones est activée par les gonadotrophines adénopituitaires et non par l'influx nerveux.

Caractéristiques morpho-fonctionnelles des glandes endocrines. Section périphérique du système endocrinien : composition, connexion avec l'hypophyse. Principes de régulation de l'activité des glandes endocrines hypophysaires dépendantes et indépendantes de l'hypophyse.

Conclusion

Aujourd'hui, les médecins ont suffisamment bien étudié le système endocrinien pour prévenir les troubles des fonctions hormonales et les guérir. Mais les découvertes les plus importantes restent à venir. Il existe de nombreuses zones vierges sur la « carte » endocrinienne du corps qui intéressent les esprits curieux.

Les hormones humaines sont conçues pour contrôler les fonctions du corps, les réguler et les coordonner. Leur travail définit notre apparence, l'activité et l'excitation se manifestent. Ces produits chimiques biologiquement actifs ont un effet puissant sur l’ensemble du corps grâce à leur interaction avec les récepteurs. Les hormones transmettent des informations d'un organe à un autre et connectent un organe à un autre. Cela vous permet d'atteindre un équilibre dans le fonctionnement de tout le corps.

Les hormones sont ce qui vous rend spécial et différent des autres. Ils déterminent vos caractéristiques physiques et mentales, si vous deviendrez grand ou pas très grand, gros ou mince. Nos hormones affectent tous les aspects de votre vie, du moment de la conception jusqu'à la mort. Ils affecteront votre croissance, votre développement sexuel, la formation de vos désirs, le métabolisme du corps, la force musculaire, l’acuité mentale, le comportement et même votre sommeil.

Littérature:

    Mécanisme d'action des hormones 1976

    Agazhdanyan N.A. Katkov A. Yu. Réserves de notre corps 1990

    Tepperman J., Tepperman H. Physiologie du métabolisme et du système endocrinien. 1989

dire aux amis